ca june 2016 eng xaam.in

68
8/16/2019 CA June 2016 Eng Xaam.in http://slidepdf.com/reader/full/ca-june-2016-eng-xaamin 1/68

Upload: rravat111

Post on 05-Jul-2018

228 views

Category:

Documents


0 download

TRANSCRIPT

Page 1: CA June 2016 Eng Xaam.in

8/16/2019 CA June 2016 Eng Xaam.in

http://slidepdf.com/reader/full/ca-june-2016-eng-xaamin 1/68

Page 2: CA June 2016 Eng Xaam.in

8/16/2019 CA June 2016 Eng Xaam.in

http://slidepdf.com/reader/full/ca-june-2016-eng-xaamin 2/68

Page 3: CA June 2016 Eng Xaam.in

8/16/2019 CA June 2016 Eng Xaam.in

http://slidepdf.com/reader/full/ca-june-2016-eng-xaamin 3/68

ContentsStrategy   7

  Prelims Countdown: Strategy to prepare in next 90 days

Debate  222

  Does Aadhar Invade Privacy to Achieve FiscalConsolidation?

Superfast Revision Series  229

  Supplement : Art & Culture

  100 Practice MCQs on Art & Culture

 Articles  15

Historical Personality

  Dr. Ambedkar : A Critical Review

Political / Constitutional Issues

  The Reservation Conundrum

  Rajya Sabha : Indispensable Institution of Indian Democracyand Federalism

Social Issues

  Ugliest face of gender based violence : Acid Attacks

Economic Issues

   Aviation Industry : Adding Wings to Inclusive Growth

  Exit Policy in India : Welcome, But No Goodbye!

  Fourth Industrial Revolution : The Next Big Thing!

International / Global Issues

  Understanding the U.S. Presidential Elections

Science and Tech Articles

  Genetically Modifed Crops : Gene Revolution and Beyond   Anti-Microbial Resistance : A New Challenge

Environment and Ecology

  The Green Tribunal : Watchdog for Environment

Current Affairs  67

   Most Important News Events

  Constitutional & Administrative Updates

  Economic Scenario

  International News Events

  India-World Relations

  Science & Technology

  Environment & Ecology

  Social Issues

  Other National News

  States Scan

   Art & Culture

  Sports News

  In News

Inspiration  149

  Topper’s Interview : Aman Mittal (IAS, UPSC CSE 2014)

Prelims Section 153

  PT Express : Quick GS Recap

   Map Work

To the Point 163

  What, Why, How, Where, When of Important GS Topics

Mains Section  173

  Potential Q & As : Based on Current Affairs

   Academic Vitamins : Gist of EPW, Yojana, Kurukshetra

  Emotional Intelligence: Moving beyond IQ (Ethics Paper)

Essays  210

  The paths to glory lead but to the grave

   Multinational Corporations: Savior or Saboteur 

   Announcement of Essay Competition

Interviews  215

  Preparing for the Personality Test (Part-1)

   Mock Interview & its evaluation

Page 4: CA June 2016 Eng Xaam.in

8/16/2019 CA June 2016 Eng Xaam.in

http://slidepdf.com/reader/full/ca-june-2016-eng-xaamin 4/68

Team DrishtiDisclaimer

  The views expressed in the articles in

this magazine are of the individual

writers. It is not necessary that the editor

or publisher shares the same viewpoint.

It is our endeavor to include articles

from writers believing in diverse

ideologies so that our readers can

benet from the diversity of views on

any subject.

  The information, news and facts

published in this magazine have been

duly veried and cross-checked. Even

then, if any information or fact is foundto have been published incorrectly, the

publisher, editor or printer cannot be

held responsible for any loss or damages

accruing to any specic person or

institution.

  We believe that the articles published

in this magazine have been written

originally by the writers to whom they

have been attributed. If any matter of

copyright violation comes up then thewriter would be responsible.

  All disputes would be subject to Delhi

 Jurisdiction only.

   Copyright: Drishti Publications, all

rights reserved. No part of this magazine

may be reproduced, stored in a retrieval

system or transmitted in any form by

any means, electronic, mechanical,

photocopying, recording or otherwise,

without the prior written permissionof the publisher.

 June 2016 Monthly Year 1  Issue 1

 Printed by M.P. Printers, B-220, Phase-2, Noida, Uttar Pradesh and Published for Owner, Printer & PublisherVikas Divyakirti, 641, First Floor, Dr. Mukherjee Nagar, Delhi-110009. Editor-in-Chief: Vikas Divyakirti

Contact for Advt. & Business:

Ajay Karakoti (Business Head) (0-8130392355)

Contact for Subscription:

Narender Pratap (0-8130392351)

 z Editor-in-Chief & CMD : Dr. Vikas Divyakirti z Chief Executive Ofcer: Shivesh Mishra

 z Advisory Board

  D. Kumar, Kumar Gaurav, Akhil Murti, Rajesh Mishra, NishantShrivastava, Ritesh Jaiswal, Sourabh Chaturvedi, K.P. Dwivedi.

 z Executive Editor

  Neel Parmar 

 z Editorial Team

  Dr. Vikas (Editor- Views section), Abhishek Mishra (Editor- News),Rabmeet Kaur, Dr. Deepshikha, Abhishek Gautam, Sarmad Wani,Chandra Bhan Singh, Prerna Priya, Debabrat Gogoi, SandeepVerma, Aniruddha Krishna, Gaurav Bana, Neha Saini, Sonu Bura

 z Typesetting and Designing

   Mohd. Sajid Saif, Jitender Ruhela, Vivek Kumar, Anil Kumar,Poonam Saxena, Pankaj Gupta, Sachin Pal, Rajo Kamti,Shabana Malik

 z Web Section

  Narender Pratap, Avinash Kumar, Durgesh, Geeta Pal, AbhishekKumar, Anu Raj

 z Managerial Support

  Mohd. Aftab Alam, Ekta Kalia, Ajay Karakoti, Gopal Rai,

Ravishankar Shukla, Rajesh Dhasmana, Ajay Sharma, Arun Singh,Pooja Sharma, Mohit Walia, Abhishek Singh, Nitesh Kumar Jha, Mohit Mishra

 z Day-to-day Support

   Mohit Pandey, Kundan Kumar, Gajender, Ravi Kumar, BhaveshGiri, Mahesh Kumar, Sunny, Dilip Tiwari, Vijay Kumar, AmitKumar, Vikesh Kumar, Raju Bera, Vijay Kumar, Panchanan

 Mishra, Ankit Yadav, Punit Kangda, Rahul Kumar, Manoj Kumar, Avinish Srivastava, Salman, Bhanu Pratap, Md. Shakeel, SurenderRai, Naveen Kr. Shukla, Anshul Tiwari, Amit Kr. Ruhela, AnkurDwivedi, Deepak Pal, Rishabh Kumar, Sahil, Subhash Kumar,Ram Surat Yadav, Raju Verma, Manish Kumar, Deepak Kamti,

Kishan Kanojia, Gyan Prakash Maurya

Contact us

For suggestions/feedback, Contact-

 Executive Editor 

Drishti Current Affairs Today

Drishti Publications,

641, First Floor, Dr. Mukherjee Nagar, Delhi-110009

 Phone: 87501 87501, Whatsapp: 8130392355

 Email: [email protected]

This magazine is a result of a team collaboration. We express our gratitude to all our team members. Apart from the given names,several freelance writers have also made important contributions whichhave been acknowledged alongside their articles.

Page 5: CA June 2016 Eng Xaam.in

8/16/2019 CA June 2016 Eng Xaam.in

http://slidepdf.com/reader/full/ca-june-2016-eng-xaamin 5/68

Dear Aspirants,

Of the many questions posed by life, the central one continues to be about the aim of our life. To a certain age,this question may remain unimportant for all of us; and for some, it never becomes a question at all. But anyone witha reective and curious mind is bound to be confronted by this question at a certain stage of life, especially when his/her basic needs have been fullled. I guess many among you have also faced this question in one form or other.

Many philosophers have tried to grasp the essence of life. A few have approached this question quite nihilistically.They say that life is nothing but the result of a momentary accident and will simply cease to exist one day. Germancritic and poet Heinrich Heine is believed to have said that “Sleep is lovely, death is better still, not to have been born is ofcourse the miracle”. Even our own beloved poet Mirza Ghalib has written “Duboyaa mujhko hone ne, naa hota mai to kyahota” (My being defeated me/Had I not been, what would have been?).

On the other hand, many thinkers have propagated the idea of hedonism which means the purpose of life is toenjoy maximum possible pleasures. A few philosophers (like Charvaka and Bentham) have even said that all pleasuresare of equal importance. It means that if someone seeks to satisfy oneself with material pleasures like tasty food,consumption, or a lavish life style and someone else seeks satisfaction from social service and helping others, thenwe can not claim that the one is better or worse than the other. Both are seen as seeking pleasure as per their natureand this only is the essence of life.

However, the truth is that self-centred pursuit of pleasure can never help in grasping the meaning and essenceof life. So the question which continues to stare us in face is that then, what needs to be done?

We know that all of us have different capabilities. Modern Psychology refers to it as ‘aptitude’. One way to comecloser to understanding one’s aim of life is developing one’s capabilities to the maximum and using them for betterment

of self as well as the society which we live in. I assure you, the pleasure which one feels after contributing selesslyto society cannot be equalled by any other, especially when our efforts contribute towards bringing a smile on thefaces of those who, caught between the brute forces of history, have been oppressed and downtrodden for centuries.Even if we become capable of changing even ten such lives in our lifetime, it will bring so much satisfaction that eventhe most sought after pleasures of the world will seem insignicant before it.

And, in Indian context, the opportunity to live such a golden life is most easily available to those who are successfulin civil services or politics. Their each and every decision has the capacity to positively affect millions of lives. It hasso many times been witnessed that a transfer of or mistreatment with an ofcer is followed by strong protests fromcivil society and becomes a sensitive media issue. It is only because people still look at them as their saviour.

So come on, concentrate all your energies and direct them towards fullling your dream. And your dream shouldnot merely be a quest for a post or position, but must also include the belief in the idea of change you can bring about

while being on that post. Finally, all our endeavors should be directed towards leaving this world a bit more beautifulthan it was at the beginning of our journey called life.

Look at your next three crucial months of preparation as a journey towards that dream. And we, the team ofDrishti Current Affairs Today shall always be there, striving hard at our own level, to help make your preparation alittle less hectic, much more organized and a bit easier.

With Best Wishes

 

(Dr. Vikas Divyakirti)

Editorial

Page 6: CA June 2016 Eng Xaam.in

8/16/2019 CA June 2016 Eng Xaam.in

http://slidepdf.com/reader/full/ca-june-2016-eng-xaamin 6/68

Page 7: CA June 2016 Eng Xaam.in

8/16/2019 CA June 2016 Eng Xaam.in

http://slidepdf.com/reader/full/ca-june-2016-eng-xaamin 7/68

 Just studying for hours and hours for days, months

and years is not enough. That is only hard work.

You need ‘strategy’ to prepare for the examination

to do ‘smart work’. Strategy is the guiding light of

your preparation; it is the software driving your

hardware (read hard work). Your strategy will denethe direction of the volume of work you do to prepare

for the exam. Lack of strategy or wrong strategy or

unclear strategy will cost you big in the UPSC CSE;

the examination is already the greatest (read

toughest) in India, don’t complicate it by assuming

that only hard work will see you conquer it.

You need strategy for all verticals: Preliminary,Mains and Personality Test. How to prepare for eachof the three connected-yet-separate levels and how

to change gears as the next level of exam is near?

How to study a material so as to optimize it for allthe three stages as far as possible? How to prepare in the rst few months preceding the Preliminary exam?

How to study (and what not to study) when you have just fteen days to appear for the Personality test? You

also need strategy as a horizontal input that will inform your preparation style, study hours, how you study,

what you study (and don’t), which material to consult, how you spend your time with hobbies and interests,what company you live in and so on.

Remember, one leak will sink a ship. You should know where your ship is going

or you will reach nowhere. You should know if there is an iceberg coming your way

to sink your Titanic. You should know the wind and weather as far as possible.UPSC is an ocean and you are competing with half a million other candidates

who are charting their path in the ocean cutting the waterways with sheerhard, smart work. What will differentiate you from others? How do youlearn from your experiences (hint: experience is the name we give to our

mistakes)? How clearly you see your goal? Are you relying blindly on

formulistic coachings, books, material? How well you know yourself and

how have you tuned your preparation to optimize your strengths and

build on your weak points? You see there are so many angles to your

preparation. You should bind all the exam necessities within a strategy.

Remember, the UPSC is following a strategy to choose a civil servant thatthe Government of India thinks is t for the services. You should have a

strategy to meet the expectation.

We hope we have been able to hammer the importance of strategy in

your head. In every issue of the magazine, we aim to cover various strategic

dimensions for the UPSC CSE to fortify your preparation. Arguably, this is

the most important part of the magazine. It is a do-or-die situation for you:

UPSC CSE examination. That is how you should think.

“It is our attitude more than our aptitude that determines our altitude.” And

it is our ‘strategy’ that determines the tone and tenor of our attitude. WE SEEA CIVIL SERVANT IN YOU. To begin with, believe in yourself, that is strategynumber one. All the best!

Page 8: CA June 2016 Eng Xaam.in

8/16/2019 CA June 2016 Eng Xaam.in

http://slidepdf.com/reader/full/ca-june-2016-eng-xaamin 8/68

Strategy

8 || Drishti Current Afffairs Today || June 2016

Dear Aspirants,

There are hardly 90 days left forCivil Services (Prelims) Examination,2016. As the clock is ticking, youranxiety and nervousness regardingyour Prelims preparation must be onthe rise as well. Questions like whatto read, what not to read, how to read,how much to read, how to revise sucha vast syllabus etc. must be giving younightmares. Surely, you might bepreparing from past one year; still youmay not be condent enough for theupcoming exam. Many aspirantsmight be in a state of confusion at thisstage. Not being able to complete the

whole General studies syllabus, failingto recall the learnt facts, not being ableto revise properly, lack of condenceetc. ultimately leads to irritation andnegative thoughts disturbing anaspirant’s mind. This is the time whenyou should be positive and condentbut the maze of General Studies takesyou to the path of negativity andconfusion. Therefore, at this point oftime you need a special kind of strategyto prepare General Studies which can

help you clear the PreliminaryExamination.

Every serious CSE aspirant hasher own strategy according to whichshe prepares. It is also rather inevitableto have an individualistic strategybecause there cannot be one t-for-allapproach in CSE. This is because ofthe diverse nature, needs, potential,background, of the CSE aspirants.Different individual may have

Prelims Countdown:

Strategy to prepare in next 90 days

different approach towards this exam.A student with humanities backgroundmay be uncomfortable attemptingscience questions. On the other hand,aspirants with engineering background

might be weak in economics or nddifculty learning factual information.Therefore, the purpose of this articleis not to give you any quick pill. It isto help you prioritize your preparationbetter and hence improve your chancesfor a better score that will help youclear the exam.

Structure of Preliminary

Examination

It is desirable to rst understandthe structure and syllabus of thepreliminary examination as organizedby Union Public Service Commission.As you know, Prelims exam comprisesof two objective papers of 200 markseach. Paper-1 is of General Studies andPaper-2 is of what is commonly knownas CSAT (Civil Services Aptitude test).Further, the merit of the exam isdecided by the marks secured inGeneral Studies paper only and does

not takes into account the markssecured in CSAT paper. This isbecause, CSAT has been madequalifying in nature i.e. one has tosecure minimum of 33% marks in

Paper-2 to get their Paper-1 evaluated.

General Studies Paper-1

(Syllabus)

  Current events of national andinternational importance.

  History of India and IndianNational Movement.

  Indian and World Geography –Physical, Social, Economic

  Geography of India and the

World.

  Indian Polity and Governance– Constitution, Political System,Panchayati Raj, Public Policy,Rights Issues etc.

  Economic and Social DevelopmentSustainable Development,Poverty, Inclusion, Demographics,Social Sector initiatives, etc.

  General issues on EnvironmentalEcology, Bio-diversity and Climate

Change – that do not requiresubject specialization.

   General Science.

Therefore, your success in thepreliminary exam depends upon yourperformance in Paper-1 i.e. Generalstudies but one also cannot afford tocompletely ignore Paper-2. Needlessto say that more importance shouldbe given to General studies incomparison to CSAT.

The Preliminary Exam (PT) is a rejection process. Lakhs of aspirants appear every year and only few thousandclear it. It is easy to get rejected here unless you are truly prepared.

Team Drishti

Page 9: CA June 2016 Eng Xaam.in

8/16/2019 CA June 2016 Eng Xaam.in

http://slidepdf.com/reader/full/ca-june-2016-eng-xaamin 9/68

Strategy

Drishti Current Afffairs Today || June 2016 || 9

After going through the Prelimssyllabus, the basic feeling that everyaspirant might have is what needs tobe done to qualify this crucial stage ofexam, what all is to be studied, howmuch to study and how to study?

Which section of General Studiesshould be focused more and whichsection less? What should be theaverage number of questions that oneshould attempt in both the papers?What is the effect of negative markingon the result and how to avoid negative

marking?

If we closely look at the cut-offtrends in recent years, particularlyafter CSAT was made qualifying, wend that if in Paper-1 60-65 (net)

questions are correctly attempted andif Paper-2 is qualied, then it is notdifcult for anyone to clear thePreliminary examination.

Civil Services PreliminaryExamination Cut-off (2011-2014)

Category 2011 2012 2013 2014

General 198 209 241 205

OBC 175 190 222 204

SC164 185 207 182

ST 161 181 201 174

PH-I 135 160 199 167

PH-II 124 164 184 113

PH-III 096 111 163 115

While, the ofcial cut-off for 2015Preliminary exam is not known yet,

experts believe it to be in between the

range of 110-120 for the general

candidates. On this basis we can

expect that the cut-off for 2016 exammight lie in the range of 110-120. But,one should also remember that CSAT

is now just qualifying and the levelof GS preparation of candidates will

be more rigorous in the changedscenario. Therefore, the cut-off of 2016

is more likely to rise, anywhere in the

range of 120-125. It must also be taken

into consideration that cut off depends

on the level of toughness of the paper

as well. So if the paper is easy, cut-offmight rise and vice versa. However,

if you’re able to score anywhere

between 125-130, i.e. if you’re ableto correctly attempt 60-65 questions,

you can consider yourself safe to clear

the rst hurdle of CSE.

What to read and

how much to read?

Now the question is what needsto be done to be able to correct these60-65 questions; what should be readand what can be left. For this, you needto rst understand that preparation isnot about reading all the topicsmentioned in the syllabus but also

analyzing the previous years’ papersto determine which section has more

likelihood to be asked and which hasless. If the preparation is channelizedaccording to this, then the chances ofsuccess, no doubt, increase.

Although, one must not solelydepend upon this approach since

every year the number of questionsasked from a particular section alsodiffer. For example, if we talk aboutthe Current affairs section, from 2011to 2014 on an average 5 questions wereasked. But 2015 paper saw a quantumshift where 16 questions were askedfrom this particular section. Therefore,it is difcult to predict how manyquestions can be asked from anysection but going by the trends andthe nature of questions asked, a rough

strategy can be devised.

Topic 2011 2012 2013 2014 2015

Averagenumber ofquestions

asked

History of India and IndianNational Movement

13 20 16 19 16 17

Indian Polity andGovernance

10 21 17 10 13 14

Indian and WorldGeography

16 17 18 20 18 18

Environment, Biodiversityand Sustainabledevelopment

17 14 14 20 12 15

Indian Economy, economicand social development

22 14 18 11 16 16

General Science 16 10 16 12 9 13

Current events of nationaland internationalimportance

06 04 01 08 16 7

Total 100 100 100 100 100 100

When we micro-analyze theprevious years’ papers, we nd thatevery section has some special sub-sections as well from where thelikelihood of questions being asked ismore. For example, from History,maximum number of questions aregenerally asked from Modern Indiaand Art & Culture part. So, if you wishto leave the Ancient and Medieval

History part, it won’t affect much.Similarly, from General Science section

maximum questions are asked fromBiology and new technologicalinnovations part and less number ofquestions come from Chemistry orPhysics part, which again you canafford to skip. Likewise we can deducefor other sections as well. However,it is important here to state that this

Page 10: CA June 2016 Eng Xaam.in

8/16/2019 CA June 2016 Eng Xaam.in

http://slidepdf.com/reader/full/ca-june-2016-eng-xaamin 10/68

Historical Personality 16 l  Dr. Ambedkar : A Critical Review 

Political / Constitutional Issues 21

 l  Te Reservation Conundrum

 l  Rajya Sabha : Indispensable Institution of Indian

Democracy and Federalism

Social Issues 32

 l  Ugliest face of gender based violence : Acid Attacks

Economic Issues 37

 l  Aviation Industry : Adding Wings to Inclusive Growth

 l  Exit Policy in India : Welcome, But No Goodbye!

 l  Fourth Industrial Revolution : Te Next Big Ting!

International / Global Issues 49

 l  Understanding the U.S. Presidential Elections

Science and Technology Articles 54

 l  Genetically Modified Crops : Gene Revolution and Beyond

 l  Anti-Microbial Resistance : A New Challenge

Environment and Ecology 62

 l  Te Green ribunal : Watchdog for Environment

 ARTICLES

Page 11: CA June 2016 Eng Xaam.in

8/16/2019 CA June 2016 Eng Xaam.in

http://slidepdf.com/reader/full/ca-june-2016-eng-xaamin 11/68

16 || Drishti Current Afffairs Today || June 2016

Dr. AMBEDKAR – A Critical Review Sarmad Wani

Born in a class considered low and outcaste, the boy who suffered bitter humiliation became the rst

 Minister of Law in free India, and shaped the country’s Constitution. A determined ghter, a deepscholar, humane to the core.

Background

Dr. Bhimrao Ramji Ambedkar (14April 1891 – 6 December 1956) wasborn in the military cantonment townof Mhow in the Central Provinces (now

in Madhya Pradesh). He was the 14thand last child of Ramji Maloji Sakpal,a ranked army ofcer at the post ofSubedar and Bhimabai MurbadkarSakpal. Ambedkar was born into apoor, low Mahar (Dalit) caste, whowere treated as untouchables andsubjected to socio-economicdiscrimination.

He completed his graduation from

Bombay University , M.A in Economicsand other subjects from Columbia

University, doctorate from LondonSchool of Economics.("The problem ofthe rupee: Its origin and its solution”,

his doctoral thesis was one of the rstcritical analysis of impact of colonialpolicies in India).

Inspirations

  Buddha was Ambedkar’s foremost

inspiration. He was also inspiredby the writings of Kabir.

  He was inuenced by the ideasof  John Dewey , (the pragmatic

American and his teacher) on

democracy. The Fabian Edwin R. A.

Seligman had considerable impacton his thought. He often quoted

Edmund Burke, the conservative

thinker of Britian and was inspired

by his ideas on constitutionalism.

Ambedkar’s notion of libertyalso comes close to that of T.H.

Green.

Personality

Fondly called as Babasaheb, B.R.

Ambedkar was a multidimensional

personality– a suave politician, a

restless reformer, a succinct journalist,

a sagacious lawyer, a deep thinker andan energetic activist.

  The methodology he used is very

scientifc and rational rather than

speculative.

  Ambedkar associates as well as

disassociateshimself from the grand

political streams such as liberal,

radical or conservative .

  He was inuenced by the ideas of

modernity such as urbanisation,

industrialisation, liberal education,and also by values of tradition like

Buddhism.

Timeline

Page 12: CA June 2016 Eng Xaam.in

8/16/2019 CA June 2016 Eng Xaam.in

http://slidepdf.com/reader/full/ca-june-2016-eng-xaamin 12/68

Articles

Drishti Current Afffairs Today || June 2016 || 17

  Ambedkar was a radical  in hisphilosophy of the annihilation ofcaste and a liberal in philosophyof individualism.

Ambedkar’s

views on HinduismAmbedkar was one of the severest

critics of Hinduism (which he equatedto Brahmanism) of his time He saidthat Hindu is not a community, it is agamut of castes competing with eachother without a binding collectiveconscience. Contrarily, there are strongbonds of unity within a caste whichpermeate everyday life of an individual

such as food habits, marriage,

commensality, rituals, etc. And thiswas the reason why India was not yeta nation.

Ambedkar’s views on Caste

Babasaheb's analysis of the originof untouchability and his action plansfor its eradication were different fromthe approach and practice of the casteHindu social reformers. Whatdistinguished Babasaheb from the

other social reformers was that helooked at the problems of the Dalitsfrom below, from the vantage pointof the deprived and oppressed.Ambedkar mentions in his book

FAMOUS QUOTES

  “Though, I was born as a Hindu, I solemnly assure you that I won’t dieas a Hindu.”

“Hindus claim to be a very tolerant people. In my opinion this is a mistake.On many occasions they can be intolerant and if on some occasions theyare tolerant this is because they are too weak to oppose or too indifferent

to oppose. This indifference of the Hindus has become so much a part oftheir nature that a Hindu will quite meekly tolerate an insult as well as awrong... With the Hindu Gods all forbearing, it is not difcult to imagine thepitiable condition of the wronged and the oppressed among the Hindus.”

  “Make every man and woman free from the thraldom of the Shastras,cleanse their minds of the pernicious notions founded on the Shastras,and he or she will inter-dine and inter-marry, without your telling himor her to do so.” (Annihilation of Caste)

“I thereby reject my old religion, Hinduism, which is detrimental to theprosperity of humankind and which discriminates between man and manand which treats me as inferior.”

‘Annihilition of caste’, “The outcasteis a bye-product of the Caste system.There will be outcastes as long as there

are Castes. Nothing can emancipatethe outcaste except the destruction ofCaste system.”

Origin of Caste

according to Ambedkar 

He smashed the mythological

basis of untouchability and laid bare

its socio-economic roots. Caste system

came into existence as a result of the

rivalry among the Aryan tribes at a

stage when they were starting to settle

down for a stable life. The settled tribes

employed the defeated tribesmen in

war as guards against the marauding

bands. These broken tribesmen

employed as guards became

untouchables later on.

He opined that there were four

classes in Aryan tribal society–

Brahmins, Khatriyas, Vaishyas,

Shudras as can be seen in Rig Vedic

text (Note: Aryans were the indigenous

 people of India according to Ambedkar

refuting the Aryan invasion theory).

It was essentially an open class system, 

in which individuals, when qualied,could change their class, and therefore

classes did change their personnel. At

some time in the history of the Hindus,

the priestly class socially detached

itself from the rest of the body of

people using endogamy, and through

a closed door policy became a caste

by itself. Likewise, the other castes

imitated them because of psychological

insecurity and divine promises madeby Brahmins to them.

Ambedkar explains in his research

paper “The Castes in India: Mechanism,

Genesis and Development” presented

at the Columbia University in 1916 that

the practice of endogamy was

sustained within a caste using strict

customs by regulating the number of

men and women in a group through

practices like widow burning, not

allowing widow to remarry; andprovisions like celibacy in case of

widower men.

His works such as: Castes in

India: Their Mechanism, Genesis and

Development; Annihilation of Caste;

 Who Were the Shudras; The

Untouchables: Who Were They and

 Why They Became Untouchables? are

testimonies to his independent and

original thinking

Ambedkar as a Politician

  He took part in all three Round

Table Conferences held in England.

  He was one of the biggest critics of

Congress as he saw it representing

only upper caste, class consciousness.

He gave representation before Simon

Commission demanding separate

electorate for Dalits. But he had to

nally agree to a compromise with

Gandhiji under Poona Pact whereby

he gave up his demand of separate

electorate and agreed for a higher

share of seats in legislature for Dalits

(from 72 to 147).

   In 1936, Ambedkar founded the

Independent Labour Party. In the

1937 elections to the Central

Legislative Assembly, his party

won 15 seats

Historical Personality

Page 13: CA June 2016 Eng Xaam.in

8/16/2019 CA June 2016 Eng Xaam.in

http://slidepdf.com/reader/full/ca-june-2016-eng-xaamin 13/68

Drishti Current Afffairs Today || June 2016 || 21

The Reservation ConundrumChandra Bhan Singh

The recurring demands by Jats (Haryana), Patels (Gujarat) and Kapus (Andhra Pradesh) for

Other Backward Class status has again brought to the fore, the debate on Reservation and itsvarious manifestations

“So long you do not achieve socialliberty, whatever freedom is provided 

by the law is of no avail to you”

–Dr. B.R. Ambedkar

Reservation as policy of afrma-tive action intends to achieve the goalsof political, social and economic justice

for all as enshrined in the preamble tothe Indian constitution. It is also inpractice in other countries like Sweden,China, Britain and United States underthe  policy of Afrmative action. Afrmative action seeks to correctinequities prevalent in society due todiscrimination within a culture. Often,certain sections of people aredisadvantaged for historical reasons,such as oppression or slavery or dueto abstract ideas of purity and pollu-

tion. Historically and internationally,support for afrmative action hassought to achieve goals such asbridging inequalities in employmentand pay and increasing access toeducation. In India groups that havesuffered from social disabilities relatedto their birth have been brought underthe ambit of the reservation policy thatseeks to mitigate if not eradicate theeffects of such injustice. The article is

an attempt to develop an understandingof reservation, its intended goals,achievements and lacunas in policymaking. The inequalities associatedwith birth have origins in the evolutionof the present day Indian society.

Social Stratifcation inIndian Society: Evolution

The evolution of Indian societyover ages has resulted in functional

differentiation and segregation ofsociety, into exclusive groups basedon birth which have maintained theiridentity based on distinctive charac-teristics such as occupation, foodhabits, residential pattern etc. TheVarna system developed in the later

Vedic age and by the age of Buddhathese varna distinctions became rigidmaintaining laws of exclusivity interms of marriage and dinning. Latercastes and jatis developed out ofoccupational groups and their exactrelationship with the brahmanicalVarna system is not clear. Thedevelopment of Varna, caste and jatiled to vertical stratication of thesociety and consequently thesubjugation of those placed in the

lower strata by the higher strataoccupants began and continued forages. Reservation seeks to correct thishistorical injustice by trying to bringthe disadvantaged and backwardsections of the society on an equalpedestal with the other sections.. It

would also be relevant to discuss thegenesis of present crisis with respectto reservation generated by demandsfrom various group (Jats in Haryanaand Rajasthan, Patels in Gujarat andKapus in Andhra Pradesh)(refer Box1

 for details).

Reservation as a Principle

Afrmative action is based onthree principles: compensation orreparation for historic injustice, the

Page 14: CA June 2016 Eng Xaam.in

8/16/2019 CA June 2016 Eng Xaam.in

http://slidepdf.com/reader/full/ca-june-2016-eng-xaamin 14/68

32 || Drishti Current Afffairs Today || June 2016

The ugliest face of gender based violence - Acid Attacks

Prerna Pr iya

 A deliberated assault on woman refectingthe horrid face of patriarchy!

“There is one universal truth, applicableto all countries, cultures and communities:violence against women is never acceptable,never excusable, never tolerable.”

Ban Ki-moon

Laxmi’s life overturned at the

tender age of 15 when Nahim Khan

(brother of her friend) caused havoc

by attacking her with the so called

‘modern’ weapon of revenge – Acid.

Laxmi, along with hundreds of other

victims has witnessed the magnitude

with which this form of attack affects

one’s life. In recent times, we are

witnessing a considerable rise in

victimization of women, violenceagainst them is aimed at silencing and

controlling them. According to

Convention on the Elimination of All

Forms of Discrimination Against

 Women (CEDAW), gender based-

violence is directed towards a womanonly because of the fact that she is a

Woman. Of the various forms of

violence, acid attack is one of the most

gruesome. Acid throwing, also termed

as Vitriolage is dened as the act ofthrowing acid onto the body of another

with the intention to disgure, maim,

torture or kill the victim.

A person’s face may not be critical

to survival but it is an essential element

of social survival, this is what is

targeted in acid attacks. These attacksare the result of a deeply inculcated

discriminatory attitude. Girls aredying or being disgured almost every

day in vast numbers due to acid

attacks. While some struggle with

wounds inicted on them by the

corrosive acid, few unfortunate ones

sadly succumb to it.

According to researchers and

activists instances of acid attacks have

been reported all over the world, but

particularly endemic to South Asian

countries with Bangladesh, India,

Pakistan, Cambodia having the

distinction of highest number of cases.

In India, acid attack is not restricted

to a particular race, religion or

geographical location but it does have

a gender dimension.

 Acid attack brings to surface the

underlyingmisogyny (hatred of women

or girls) a result of the patriarchal

mindset. Women in contemporary

society are valued on the basis of their

 physical appearance. Acid attack

specically targets the visual aspect of

their being and leaves them with the

trauma of social exclusion for eternity.

The consequence of an acid attack is notsomething which one can physically

move on from or efcaciously hide.

Reasons for Acid Attacks

Acid attack is a mirror of gender

discrimination and inequality faced

by women. Let us now delve into the

varying underlying causes behind this

horrendous form of crime.

Unrequited Love

Women in India are subscribedto a patriarchal mindset where they

are expected to smile demurely andpassively accept the declarations of(unwanted) love. The most commoncause behind attacking women is theirexplicit disapproval of marriage orlove proposals and at times evensexual advances. Some men, enragedover their ‘unrequited love’ (one sidedaffair, not openly reciprocated), aftertaking offence at being spurned, resortto acid as a retributive weapon onlyto teach the girl a lesson. Low levels

of education, misguided portrayal ofmen and women by media where thegirl always gives in to her resistanceafter persistent efforts by the boy towoo her, nests the idea that womanare to be acquired. A girl’s refusal toan advance is taken to be supercial,only to reach this horric end.

Cultural Causes

Such attacks stem from not just apatriarchal mindset, but also from

class inequalities, revenge, jealousy,etc. The ‘Objectifcation’ of women isan attitude which mirrors the idea ofwomen as ‘possessions’. Victims oftennarrate as how right before the attack,the attacker said that if he cannot‘possess’ them, no one else should.Traditionally in India, men have thedecision-making power and womenare bound to obey their decision. Thefeeling of dishonour which a man feels

Page 15: CA June 2016 Eng Xaam.in

8/16/2019 CA June 2016 Eng Xaam.in

http://slidepdf.com/reader/full/ca-june-2016-eng-xaamin 15/68

Drishti Current Afffairs Today || June 2016 || 37

 Aviation Industry : Adding Wings toInclusive Growth

 Amit kumar Singh

Connectivity has been a vehicleof prosperity since time immemorial.Initiatives like Silk Route in the ancienttimes and more recent ones such asOne Belt One Road have been at thecentrestage for almost all humancivilizations aspiring prosperitythrough connectivity.

However with changing times,mode of connectivity too has rapidlychanged giving way to aviation whichhas emerged as the most efcient modeconnecting economies and cultures.

In Indian context, major thrustand our understanding of connectivityhas been limited to road and rail withthe aviation sector being largelyignored. However, with PrimeMinister's ambitious 'Make In India' initiative with aviation as one of thesectors, it has gained currency.

 Why Aviation Industry acrossthe globe is eyeing India?

(a) India has an ideal geographiclocation between Eastern and Westernhemisphere which can provide break

 journeys to long route operations.

(b) 300 million strong middle classand hence a potential market to theirservices. For Instance: The totalnumber of domestic tickets sold in2014-15 were 70 million. If 300 million

strong middle class ies even once,the annual sale of tickets will surgefrom 70 million to 300 million.

 Why we need a strongAviation Industry?

(a) The development of this sectorhas multiplier effect that positivelyimpacts output and employment ofother sectors to a very impressive scale.

(b) If airfare on regional routes

are brought down, affordable yingby masses will lead to productivityenhancement, spending lesser time intravelling and more on skilldevelopment.

(c) Incidents such as BorderSecurity Force aircraft crash lastDecember in Dwarka (Delhi), whichclaimed ten lives of BSF Jawans merelyfor the reason of engine failure whichcould have been curbed, if we haveMaintenance Repair and Overhaul(henceforth, MRO) facilities in place,which is a critical component ofaviation industry.

(d) Growth of aviation industryaccompanies investments in form ofForeign Direct Investment (FDI), invarious sectors of the economy, suchas tourism etc. This will also increaseemployment for semi-skilled andunskilled labour due to positivespillover effect. It is to be noted thatsemi-skilled and unskilled labour is acause of concern due to dwindlingviability of agriculture as a sustainedsource of livehood and inability ofmanufacturing sector to absorb sucha workforce.

However for aviation industry toplay its due role in the ambitious MakeIn India initiative, we need to have acloser look at the sector with somestructural interventions  awaiting

attention.

Maintenance, Repair &Overhaul (MRO) Facilities

  India's airlines spend approx ` 5000crores on MRO annually. However,almost 90% of this amount goes toMRO units in countries like SriLanka, Singapore, Malaysia, UnitedArab Emirates and Indonesia. Thisis due to the fact that MRO facilities

in these countries offer services atlower cost with higher technicalexpertise and take comparativelyless time.

  Presently commercial airlines inIndia operate more than 400 planes;the number of privately ownedplanes is even higher. Time boundMRO facilities is necessary to ensuresafety of these aircrafts.

  According to a report by GlobalConsulting rm, KPMG, (March2014) by 2020 India's eet size woulddouble and hence domestic MROindustry will become more criticalin terms of security of aircrafts andas a source of earning foreignreserves.

  Union Budget  (2016), proposeseasing tariff barriers (custom duty,countervailing duty) on tools, toolkits, service parts involved inmaintenance, repair and overhaulas a positive policy intervention.

  However one of the major bone ofcontention is high service tax onheavy maintenance visit, that makesIndian vision of becoming an MROhub unviable.

Service tax on heavy maintenancevisit includes locally-sourced parts,tools, equipments and spares plus theadditional charging of nearly 15% tax

on top of nal bill. Such application oftax makes maintenance, repair andoverhaul, comparatively expensive inIndia. Policy makers need to criticallyexamine the cost-benet ratio of easingservice tax norms. As reducing servicetax may deplete revenues for the publicexchequer but at the same time haspotential to attract foreign exchange thatpresently falls in the hands of countrieslike Sri lanka, Malaysia, Singapore.

Investor lobby also believes that

Page 16: CA June 2016 Eng Xaam.in

8/16/2019 CA June 2016 Eng Xaam.in

http://slidepdf.com/reader/full/ca-june-2016-eng-xaamin 16/68

Drishti Current Afffairs Today || June 2016 || 41

Exit Policy: Welcome, But No Goodbye!

Rabmeet Kaur 

The Indian economy, over the course of six decades, has moved from ‘socialism with limited entry to

“marketism” without exit’. The article takes a look at how the absence of an exit policy impacts the economy.

Since the early 1980s, the Indianeconomy has made remarkableprogress in increasing entry:dismantling of industrial licensing,dilution of public sector monopolies,privatization of few public sectorassets, considerable liberalisation of

foreign direct investment as well asreduction in trade barriers. However,while liberalisation of the economyhas given freedom of entry, a liberalpolicy towards the entry and expansionof rms can be benecial only if it isaccompanied with a rational policytowards the exit of unviable rms. Anexit policy is regarded as aquintessential cog in the wheels ofliberalization.

An Exit Policy provides a business

unit the right or ability to withdrawfrom or leave an industry. It facilitatescom-panies to close down theirbusiness, reorient their business, andrestructure their operations with theleast restric-tions and in quick timeframe. This enables the removal ofresources away from inefcient andunsustainable uses. For example, itallows employers to shift workersfrom one unit to another and/or alsoretrench excess labour. The need for

such a policy arises as a result ofmodernisation, technologyupgradation, restructuring as well asclosure of business units. It is anecessary condition for inducingcompetition and enhancing theefciency of resource use.

The legend of the Chakravyuhafrom the Mahabharata, describing theability to enter but not exit, perfectlyillustrates the workings of the 21stcentury Indian economy. The presence

NO EXIT

ENTRANCEONLY

of disproportionately large share ofinefcient rms with extremely lowproductivity in India is directly relatedto an exit problem in Indian industry.This is because a majority of these largenumbers of small and inefcient rmsshould not survive. An effective

insolvency regime saves strugglingrms when possible, or reallocatesassets of failing rms moreproductively.

Current Framework in IndiaAt present, India lacks a single,

comprehensive law to address the

several aspects related to insolvencyof an enterprise. Corporate bankruptcyand insolvency is covered in a complexof multiple laws, some for collectiveaction and some for debt recovery.These are:

(a) Companies Act, 2013 – Chapter

on collective insolvency resolution byway of restructuring, rehabilitation,or reorganisation of entities registeredunder the Act.

Adjudication is by the NationalCompany Law Tribunal (NCLT).

National Company Law

Tribunal

NCLT, a quasi judicial mecha-nism, would encompass the power

and jurisdiction of the CompanyLaw Board, the Board for Industrialand Financial Reconstruction (BIFR),the Appellate Authority forIndustrial and Financial Reconstruc-tion and of the High Court relatingto company law matters. The NCLThas not, however, been put into op-eration as yet.

(b) Companies Act, 1956– dealswith winding up of companies.

No separate provisions forrestructuring except throughMergers & Acquisitions (M&A) andvoluntary compromise.

Adjudication is under the jurisdiction

of the High Court.

(c) Sick Industrial Companies(Special Provisions) Act (SICA), 1985– deals with restructuring of distressed‘industrial’ rms.

Under this Act, the Board of

I n d u s t r i a l a n d F i n a n c i a lReconstruction (BIFR) assesses theviability of the industrial company,and refers an unviable company tothe High Court for liquidation.

   Judicial oversight over the proceed-ings before the BIFR was limitedby giving appellate jurisdiction tothe Appellate Authority forI n d u s t r i a l a n d F i n a n c i a lReconstruction (“AAIFR”).

Page 17: CA June 2016 Eng Xaam.in

8/16/2019 CA June 2016 Eng Xaam.in

http://slidepdf.com/reader/full/ca-june-2016-eng-xaamin 17/68

Drishti Current Afffairs Today || June 2016 || 45

Fourth Industrial Revolution:The Next Big Thing!

Sandeep Verma

“If you go back to 1800, everybody was poor. I mean everybody. The IndustrialRevolution kicked in, and a lot of countriesbeneted, but by no means everyone.”

- Bill Gates

Humankind is at the threshold ofa new industrial revolution driven bythe conuence of a staggering rangeof emerging technologies. The big buzzat the World Economic Forum (WEF)in Davos this year was about the‘Fourth Industrial Revolution’,described by the founder and executivechairman of WEF, Klaus Schwab, asa “technological revolution that willfundamentally alter the way we live,work and relate to one another”.

Earlier Industrial Revolutionsadvanced human progress throughnew forms of power generation, massproduction and informationprocessing. The First IndustrialRevolution began in Britain in the last

quarter of the 18th century with themechanization of the textile industry,harnessing of steam power, and birthof the modern factory. The SecondIndustrial Revolution, from the

middle of the nineteenth century tothe outbreak of World War I, waspowered by developments inelectricity, transportation, chemicals,steel, and mass production andconsumption. Industrialization spreadeven further to Japan after the MeijiRestoration and deep into Russia,which was booming at the outset ofWorld War I. During this era, factoriescould produce countless numbers ofidentical products quickly andcheaply. The Third IndustrialRevolution, beginning in early 1970s,was digital and applied electronics andinformation technology to processesof production. Mass customization andadditive manufacturing - the so-called‘3D printing’ are its key concepts, and

its applications, yet to be imaginedfully, are quite mind-boggling.

 What is FourthIndustrial Revolution?

The soon-to-happen and highlyawaited Fourth Industrial Revolution combines digital, biological  andphysical systems and will propagatea new interaction between humansand machines. Built upon the previousthree Industrial Revolutions, this willprove the rapid speed of technologicalprogress by fusing their boundaries.

The First Industrial Revolutionstarted in the 18th century with

the use of water and steam powerto mechanize production.

  The Second in 19th century usedelectric power to create massproduction.

  The Third began in the 1960s andused electronics and informationt ec hno l o gy to a u to m a teproduction.

  Now a Fourth Industr ialRevolution is building on the

third, that is, the digital revolution.In a paper on “The Fourth

Industrial Revolution: What it means,how to respond”, Schwab says thatthree things about the ongoingtransformation mark it out as a newphase rather than a prolongation ofthe current revolution — velocity,scope, and systems impact. The speedof current breakthroughs has nohistorical precedent. When compared

Fourth Industrial Revolution led by technological fusion is the emerging paradigm in industrial evolution. The article throws a searchlight on what it means and how it will impact the world community.

Page 18: CA June 2016 Eng Xaam.in

8/16/2019 CA June 2016 Eng Xaam.in

http://slidepdf.com/reader/full/ca-june-2016-eng-xaamin 18/68

Drishti Current Afffairs Today || June 2016 || 49

The US presidential election iseasily one of the most talked aboutdevelopments in the world this year.The multi-stage election jamboree lastsfor a long time spread over almost ayear which can be very confusing foroutsiders. Particularly, this year it has

been hogging more limelight as theprocess has been vitiated by theadvances made by a controversialDonald Trump who is not bound bypropriety and political correctness.Trump has been blunt, rude, racist,outspoken, extreme at times and yethe has been gaining ground. That issomething that surprises electionanalysts and presents added charm toUS elections for that reason. Onceevery four years (rst Tuesday after

the rst Monday in November), theUS citizens votes to decide their nextpresident. Here is the process of thispolitical marathon.

TIMELINE OF USELECTIONS 2016

  PRIMARIES and CAUCUSES:Public voting to select presidentialnominees for major parties starting1 Feb 2016 (Iowa caucus) to 7 June

2016 (nal 6 State Primaries) NATIONAL CONVENTION

of Parties to nalise nominee :Republicans 18-21 July; Democrats25-28 July 2016

  ELECTION DAY: Americansvote, 8 November 2016.

  PRESIDENT’S INAUGURATION:20 January 2017

More than half the battle for acandidate is about winning enough

support to gain a Party’s nomination;the election day may be the 8th ofNovember 2016 but to be declared acandidate for that fateful day, one hasto go through a long-winding processof winning the primaries and caucusesin various US states.

The Primaries and Caucuses:Fight to Secure Delegates

We keep hearing of primaries andcaucuses and how Hillary, Bernie orTrump make advances in various USstates where they are being held. It isthe starting step for selection of aparty’s presidential candidate wherethe general public vote for theirfavourite in the party they support.

There are many presidential hopefulswho belong to one of the two mainpolitical parties, Democrats orRepublican, depending on their viewsand ideas about how the governmentshould work.

In most other countries includingIndia, it is the party that decides onthe candidate. But in the United States,it is the people who declare supportfor one of the parties and choose aparty candidate from a list of

candidates competing for the positionof POTUS (President of the UnitedStates). This is where primaries andcaucuses have a role; candidates fromeach party campaign throughout thelength and breadth of US to win thesupport of party members. In eachsuch contest, a candidate has to securemajority of his or her party’s delegatesor individuals who represent theirstates at national party conventions.The candidate accumulating a majority

of party’s delegates (threshold delegatecount in 2016 is 1237 for Republicansand 2382 for Democrats) in this longprocess lasting over many monthswins the nomination of his or herparty.

Primaries  (vote by secret ballotby people appearing at the regularpolling station; used in most US states,it is paid for by the state and conductedby state election ofcials) or Caucuses (active debate and discussion for hoursby the community followed by votingin evening; usually held at town halls,school gyms and other public venues)are the two ways in which states collectparty members’ votes. Primaries couldbe closed (only registered voters of theparty can participate) or open (no voterregistration with party is needed butvoter can vote only in primary of oneparty; a controversial aspect ofelections). Before the 1970s, most stateschose delegates using caucuses butpost-1972 reforms, most states adoptedprimaries for a more inclusive andtransparent process. In 2016, justfourteen states hold caucuses.

People vote for delegates who arethe representatives who will choose

presidential and vice-presidentialnominees at the national conventionof parties (in late July 2016). Delegatesare generally party activists, localparty supporters, early supporters ofcandidates; presidential campaignscourt such people as they bring withthem a certain political backing for thecandidature. Democrats awarddelegates on a proportional basis: one-third of votes results in one-third ofdelegates and so on.

Team Dr isht i

Understanding the US PresidentialElections

Team Dr isht i

 A breakdown of the long and complex process involving Primaries,National Conventions and Electoral College

Page 19: CA June 2016 Eng Xaam.in

8/16/2019 CA June 2016 Eng Xaam.in

http://slidepdf.com/reader/full/ca-june-2016-eng-xaamin 19/68

54 || Drishti Current Afffairs Today || June 2016

Genetically Modifed Crops : GeneRevolution and Beyond

resources like soil, water, land availability are fastdeteriorating and depleting. The mounted pressureson two fronts is forcing our scientists to look beyondtraditional methods of agriculture and one hopefulbeacon seems to be the ‘’GM crop’’.

Since 2002, when GM crops were rst introducedin India as Bt cotton (which still remains the only

GM crop being commercially produced), there havebeen a series of events in support of and against theGM crops. From PILs being led by activists inSupreme Court against GM food crops to theparliamentary standing committee reports that haveshown apprehensions to the use of GM crops,moratoriums against the eld trials to the recentthumbs up being given by the Central Government,India is yet to come out decisively on the issue ofeld trials, commercialization and cropping despiteemerging as the fourth largest country to grow GMcrops in 2013.

The issue evokes concerns and hopes becauseafter Green Revolution we haven’t come across anypromising innovation that will salvage the situationin agricultural sector and guarantee food securitysustainably without any additional costs. But theenvironmental impact it may create is a worrisomefact. As is the case with every scientic blessing,this one too is a mixed bag of potential benets andunknown dangers that may arise later.

 What are genetically modifed crops?

To understand the issues surrounding it weneed to have clarity on what GM crop actuallymeans. GM crops are basically the plants used inagriculture, the DNA of which has been modiedusing genetic engineering techniques with the aimto introduce a new desired trait to the plant whichdoesn’t occur naturally like resistance to pests, dis-eases or environmental conditions, or improvingthe nutrient prole of the crop.

Since we are genetically modifying the plant,we are able to address a host of problems that

“It is better to die eating GM food instead of dying of hunger”.

This statement by Dr. Norman Borlaug, father of GreenRevolution, sums up the difcult choices that India is beingconfronted with. In the present scenario where on one hand itneeds to ensure food to its 1.2 billion plus population battlingpoverty, hunger, malnutrition, on the other hand our agricultural

Dr. Deepshikha

“The resilience we need for the future will be delivered by smart plant breeding- and that’s all GM is.” –George Freeman

Page 20: CA June 2016 Eng Xaam.in

8/16/2019 CA June 2016 Eng Xaam.in

http://slidepdf.com/reader/full/ca-june-2016-eng-xaamin 20/68

58 || Drishti Current Afffairs Today || June 2016

 Anti-Microbial Resistance : A New Challenge

Antibiotics have been used for thelast 70 years to ght infections causedby bacteria in both humans andanimals. Antibiotics either kill thebacteria or make it difcult for thebacteria to grow and multiply. The rstcommercialized antibiotic, Penicillin,was discovered by Alexander Fleming

in 1928. But the prolonged andwidespread use of antibiotics hasmade the infectious organisms(bacteria, fungi, viruses and parasites)no longer responsive to the drug,making them less effective. The threatof antibiotic resistance, i.e. whenbacteria change and become resistantto the antibiotics used to treat theinfections they cause, dates back to thetime when Fleming won the NobelPrize for his discovery and in his

acceptance speech warned of bacteriabecoming resistant to penicillin.

But according to a new report ofthe WHO, “this serious threat is nolonger a prediction for the future, it ishappening right now”. Antimicrobialresistance is now a major threat topublic health across the globe. It canaffect anyone, of any age, in anycountry. Resistance to treatment of lastresort (carbapenem antibiotics)  for

life threatening diseases has spreadacross the world, though the extent ofresistance varies. It causes prolongedsickness and increases the risk ofdeath. The problem is so severe thatit threatens the achievements ofmodern medicine. New resistancemechanisms emerge and spreadglobally every day, threatening ourability to treat common diseases. Apost-antibiotic era— in whichcommon infections and minor injuries

can kill— no longer seems to be anapocalyptic fantasy, rather a very realpossibility for the 21st century.

What is the difference betweenantibiotic and antimicrobial

resistance?

  Antibiotic resistance refers

specically to the resistance toantibiotics that occurs in commonbacteria that cause infections.

  Antimicrobial resistance is abroader term, encompassingresistance to drugs to treatinfections caused by othermicrobes as well , such asparasites (e.g. malaria), viruses(e.g. tuberculosis and HIV) andfungi (e.g. Candida).

How It Happens Bacteria can become resistant to

antibiotics through several ways. Somebacteria due to the presence ofresistance genes are intrinsicallyresistant and therefore survive onbeing exposed to antibiotics. Due to alife-cycle and generation time spanning

a few hours and days, these survivorsreplicate and form a dominant typeresulting in the development of anantibiotic resistant strain. Bacteria canalso acquire resistance. This canhappen in two ways: by sharing andtransferring resistance genes presentin the rest of the population or by

genetic mutations that help the bacteriasurvive antibiotic exposure. Once theresistance has been acquired, it canspread in the rest of the population ofbacteria through reproduction or genetransfer.

How It Spreads

Though antibiotic resistanceoccurs naturally, its development andspread is accelerated by the misuseand overuse of antibiotics in humans

and animals. When antibiotics aretaken, bacteria sensitive to it get killed,but the resistant strain survives, growsand multiplies. Poor prescribingpractices like prescribing antibioticwhen not required, incorrect choiceof medicine, wrong dosage, self-medication in countries where

Team Dr isht i

 AMR threatens our ability to effectively prevent and treat an ever-increasing range of infections caused bymicro organisms. Let’s take a look at how this happens and what can be done to minimize its emergence.

Page 21: CA June 2016 Eng Xaam.in

8/16/2019 CA June 2016 Eng Xaam.in

http://slidepdf.com/reader/full/ca-june-2016-eng-xaamin 21/68

62 || Drishti Current Afffairs Today || June 2016

The Green Tribunal: Watchdog for Environment

Neha Saini

NGT has nally started giving environment its due in India. The article takes a look at its evolution, role and challenges.

National Green Tribunal (NGT)makes news on a regular basis. Itsdecisions to safeguard environmenthave had an impact on ministries,

corporations and individuals. It wouldbe wise to pause and think aboutNGT’s efcacy. The NGT’s roleassessed here is in context of therecently held World Culture Festival onthe banks of river Yamuna by the Artof Living Foundation. The tribunalreprimanded the Ministry ofEnvironment, Forest and ClimateChange (MoEFCC), Delhi Governmentand Delhi Development Authority(DDA) for not seeking environmentalclearance for such a large scaleanthropogenic activity in anecologically sensitive region and forgranting the public land for a privateorganization’s festival. However, NGT

allowed the continuation of the eventby imposing a penalty of Rs. 5 croreon the Art of Living Foundation as‘environment compensation’.

It is in this background that wewill be evaluating the evolution ofNGT, its mandate and its effectivenessin ensuring safe and clean environment.

Evolution of NGT

Although, two major acts, theNational Tribunal Act, 1995 and theNational Environmental AppellateAuthority Act, 1997 were in existence,they were never implemented in truespirit. With increasing industrialization,

need arose for a specialized tribunal.Also, India had vowed to providespeedy judicial and administrativeremedy to pollution affected victimsat the United Nations Conference on

Environment and Development heldat Rio De Janerio in 1992.

In addition, the judiciary of thecountry in Dehradun Quarrying Case

(1988) enlarged the scope of Article21. Since then, Article 21 grants not

 just the Right to Life, but the Right toClean Environment as well. TheConstitution under Article 48-Aprovides the need to 'protect andimprove the environment and tosafeguard the forests and wildlife ofthe country'.

Again, in M.C. Mehta Vs Union ofIndia  (1987) and Indian Council forEnviro-Legal Action vs Union of India(1989), the Supreme Court oated theneed for a fast track court to provide

 justice to the victims of environmentalpollution.

Page 22: CA June 2016 Eng Xaam.in

8/16/2019 CA June 2016 Eng Xaam.in

http://slidepdf.com/reader/full/ca-june-2016-eng-xaamin 22/68

CURRENT AFFAIRS(A Compilation of Important News Events)

  1  Most Important News Events 68

  2  Constitut ional & Administrative Updates  86

  3  Economic Scenario 

96

  4  International News Events  106

  5  India-World Relations  114

  6  Science & Technology  120

  7  Environment & Ecology 

127

  8  Social Issues  131

  9  Other National News 133

 10  States Scan  137

 11  Art & Culture  141

 12  Sports News  143

 13  In News  145

Page 23: CA June 2016 Eng Xaam.in

8/16/2019 CA June 2016 Eng Xaam.in

http://slidepdf.com/reader/full/ca-june-2016-eng-xaamin 23/68

68 || Drishti Current Afffairs Today || June 2016

The historic Paris Agreement on

climate change marked a major

milestone with record 175 countries,

including India, signing on to it on the

opening day on 22nd April 2016 (Earth

Day). The world leaders also agreed

that more action within specied time

frame is needed to ght the relentless

rise in global temperatures. With theplanet heating up to record levels, sea

levels rising and glaciers melting, the

pressure to have the Paris Agreement

enter into force and to have every

country turn its words into deeds was

palpable at the U.N. signing ceremony.

  The Paris Agreement negotiated

at COP-21 in December 2015  is

the rst global accord to commit

nearly every nation to take domestic

actions to tackle climate change.

  To promote the accord, the UN

Secretary-General Ban Ki-moon

planned the signing ceremony for

April 22, Earth Day in New York.

  The agreement will enter into force

once 55 countries representing at

least 55% of global emissions have

formally joined it, a process initially

expected to take until 2020. But

following a host of announcementsat the signing event it could happen

perhaps later this year.

  China  which is the world’s top

carbon emitter, announced it would

ratify the agreement before the G20

summit in China in September 2016.

China has endorsed an aggressive

expansion of renewable energy

sources and its latest five-year

economic plan calls for generating

15% of its energy from non-fossil

fuel sources by 2020.

  The United States, the world’ssecond-largest emitter, reiteratedits intention to ratify this year, asdid Canadian Prime Minister Justin

Trudeau and the leaders of Mexicoand Australia.

  Combined together the U.S. and

China account for about 40% ofglobal emissions.

  Those that have not indicatedthey will sign include some of theworld’s largest oil producers— Saudi Arabia, Iraq, Nigeria andKazakhstan.

The Paris Agreement has been amajor breakthrough in U.N. climatenegotiations, which for years were

1  Most Important News Events

Earth Day

Earth Day is an annual event, celebrated on April 22. Worldwide eventsare held to demonstrate support for environment protection. It was rstcelebrated in 1970, and is now coordinated globally by the Earth Day Networkand celebrated in more than 193 countries each year.

Earth Hour

 World Wide Fund for Nature’s (WWF) Earth Hour is an annual globalcelebration in which people switch off their lights for one hour to show theycare about the future of our planet.

  This year’s celebrations (2016) were on 19th March from 8.30 pm to 9.30 pm.

  It began in Sydney, Australia in 2007, the number of countries taking partin Earth Hour has grown to an incredible 178 countries and territories.

  Earth Hour is not about how much energy is saved during the hour.Rather, it is a chance to put the spotlight on the issues facing the planet,and to inspire millions across the world to live more sustainably.

  Next year’s event will be at 8.30pm – 9.30pm on 25th March, 2017.

PARIS CLIMATE AGREEMENT SIGNED BY 175 COUNTRIES

Page 24: CA June 2016 Eng Xaam.in

8/16/2019 CA June 2016 Eng Xaam.in

http://slidepdf.com/reader/full/ca-june-2016-eng-xaamin 24/68

Current Affairs

Drishti Current Afffairs Today || June 2016 || 69

slowed by disputes between rich andpoor countries over who should dowhat. Under the agreement, countriesset their own targets for reducingemissions of carbon dioxide and othergreenhouse gases. The targets are not

legally binding, but countries mustupdate them every ve years.

Scientic analysis shows that the

initial set of targets that countries

pledged before Paris don’t match the

agreement’s long-term goal to keep

global warming below 2°C (3.6°F),

compared with pre-industrial times.

Global average temperatures have

already climbed by almost 1°C and

last year was the hottest on record.

Accountability

for rise in temperature

  The global 1 degree rise in

temperature was due to 150 years

of uncontrolled carbon emission by

the developed world.

While United States was responsible

for 30% of cumulative contribution,

Europe, Canada and other developed

world for 50%, China for another10% and India was responsible for

only 3%.

India raties the Paris Pact

India ratied the Paris agreementon climate change on April 22, 2016.The United Nations agreement,negotiated in Paris in December 2015,sets out a global action plan to put the

world on track by limiting globalwarming below 2 degrees Celsius andattempt to shield the world from thecatastrophic effects of climate change.In Paris, Prime Minister Modi hadintroduced the concept of climate

 justice driving home the message ofsustainable development.

Key concerns forIndia after Paris Accord

  India’s estimate of its share of

global greenhouse gas emissionssubmitted to the UN for the Paristreaty is 4.10%.

  India faces a difficult choice ofhaving to emit large volumes ofcarbon dioxide to achieve growth,while at the same time preparingto adapt to the destructive effectsof climate change.

  India will have to sharply cutemissions intensity of GDP by 2020.

  Energy, transport and infrastructure

are key areas where sound nationalpolicies are needed to cut emissionintensity.

Proactive measures by India

to combat Global Warming 

  A proposed Compensatory

Afforestation Funds Bill, 2015, to

unlock Rs. 40,000 crore of funds

for States to take up afforestationprogrammes.

  The doubling of the cess on coal in

the Budget for encouraging shift

to cleaner fuel,  and the general

policy to keep fuel prices high

using taxation are welcome, but

they must translate into funding

for green alternatives.

India must unlock middle class

investments in renewable energywith an effective grid-connected

rooftop solar subsidy programme.

In the absence of strong backing

from State governments progress

in this area has been slow.

  New buildings should also be

required to conform to energy

efciency codes in all States.

The National Electric Mobility

Mission Plan aims to put about sevenmillion electric or hybrid vehicles

on the road by 2020, but for this to

happen, the creation of

charging infrastructure

and introduction of

consumer incentives

are vital; greening

public transport bus

eets will be needed.

The success of theclimate compact will

ultimately depend on

whether rich countries

fund innovation and

open-source their

green technologies to

developing nations

like India.

Most Important News Events

Page 25: CA June 2016 Eng Xaam.in

8/16/2019 CA June 2016 Eng Xaam.in

http://slidepdf.com/reader/full/ca-june-2016-eng-xaamin 25/68

Current Affairs

70 || Drishti Current Afffairs Today || June 2016

On March 31 and April 1, leadersof 52 countries including India cametogether in  Washington DC  for the

fourth Nuclear Security Summit (NSS).NSS was held every two years since2010 and started with the recognitionof the risks posed by plutonium andhighly enriched uranium (HEU), thekey ingredients for making nuclearweapons, and aimed to secure allvulnerable nuclear material in fouryears.

Nuclear Industry Summit(NIS) 2016

  The NIS is an ofcial side event ofthe NSS. Both of these were startedin 2010 to secure vulnerablenuclear materials, break up blackmarkets, and detect and interceptillicitly trafcked materials.

During NIS 2016, hundredsof CEOs and industry leadersconferred about nalizing actionsfrom previous Nuclear IndustrySummits and address three key

areas:  l  Securing the Use, Storage

and Transport of StrategicNuclear and RadiologicalMaterials

  l  Managing the Cyber Threat

  l  The Role of the NuclearIndustry Globally

  The Nuclear Security Summit was an

initiative of President Barack Obama

to coordinate international efforts

to prevent terror organisationsfrom acquiring nuclear weaponsor material.

  This was the fourth and final

Nuclear Security Summit (NSS)

and attended by leaders frommore than 50 countries and fourinternational organisations — the

European Union, the International

Atomic Energy Agency (IAEA), theInterpol, and the UN.

Four types of specic nuclearthreats posed by terror outts

These groups could acquire anuclear weapon from the arsenalof a nuclear state.

  These terror groups acquire enoughfissile material to construct animprovised nuclear device  asthis know-how exists outsidegovernments too.

  They could acquire radioactivematerial from civilian sourcessuch as hospitals or universitylaboratories that could be mixed

with conventional explosives tomake a radioactive dispersal deviceor dirty bomb.

  Terror groups could also sabotagea nuclear facility leading to large-scale loss of lives and destruction.

Russian absence from NSS

  The strained relations with theU.S. have led to Russia skippingthe summit.

U.S and Russia are two countries thathave the largest nuclear stockpilesand their disagreements could limitthe prospects of the internationalcooperation on the issue.

  Despite the tension, cooperationbetween the two countries continuesas can be seen from the fact thatRussia took the responsibility toremove the highly enriched uraniumfrom Iran, cooperation on the NewSTART Treaty and removal of 1,300

tons of chemical weapons from Syriain recent years.

India at NuclearSecurity Summit

Prime Minister Modi attended thetwo-day Nuclear Security Summit(NSS) in Washington DC. Indiapledged a further contribution of $1million to the IAEA nuclear securityfund taking its total contribution to $2million.

  The Prime Minister underlinedthe priority India has attachedto nuclear security at home, in

terms of institutional frameworks,resources for training people, andby continuing to reflect India’sinternational obligations in nationalactions.

India was moving to safertechnologies to protect radioactivematerial, for example, the shift tothe use of Cesium 137 only in itsvitried form in medical equipmentand moving away from powder andliquid forms.

  India plans to enhance engagementwith the IAEA,  the Interpol andother international forums on theissue of nuclear security.

India‘s vulnerability toNuclear Terrorism

  India is a source of nuclear materialand a potential target of nuclearterrorism.

While India takes pride in the

security of its nuclear installations,‘orphan sources’ i.e., devices withradioactive materials outsideregulatory and security measurescould pose serious risks.

The Washington DC based NuclearThreat Initiative ranked India lowin nuclear security measures andcited corruption as a key reasonthat could compromise its nuclearfacilities.

India’s nuclear materials securityconditions could be improved bystrengthening laws and regulationsfor on-site physical protection,control and accounting, andmitigating the insider threat, andensuring protection of materialsduring transport.

Lessons from the NSS

The Nuclear Security Summit(NSS) process was initiated with

FOURTH NUCLEAR SECURITY SUMMIT (NSS)

Most Important News Events

Page 26: CA June 2016 Eng Xaam.in

8/16/2019 CA June 2016 Eng Xaam.in

http://slidepdf.com/reader/full/ca-june-2016-eng-xaamin 26/68

Current Affairs

Drishti Current Afffairs Today || June 2016 || 71

fanfare in 2010 by US PresidentBarack Obama to prevent non-stateactors,  particularly terrorists, fromacquiring nuclear material and secureall vulnerable nuclear material in fouryears. But six years and four summitslater, this goal has not been reached,despite substantial progress beingmade.

  The NSS is narrowly focused on thethreat of non-state actors acquiringnuclear material.

  The fact that the 2012 and 2014summits were held in South Koreaand the Netherlands respectively—

both US allies from the developedworld—indicates that Washington isstill not able to nd willing partnersfor its initiatives in the global South.

  The NSS process only deals withnuclear material in civilian facilitiesand not the military nuclearfacilities, which account for about83% of all nuclear material.

  The danger posed by forward-deployed tactical nuclear weapons, 

particularly by Pakistan, Russia andthe US, has not been addressed andneeds to be remedied.

 

The relative success of the NSSprocess also underlines the failureof the international community toaddress similar dangers emanatingfrom biological weapons. As there isno international regime or institution

to deal with biological weapons,they remain largely unregulated.

  For India, while its contribution tothe success of the NSS process isuseful to highlight its credentialsas a responsible nuclear state, anyinitiative on similar threats frombiological weapons and its abilityto rally others to the cause wouldenhance its credibility as a globalleader.

Achievements of NSS

  Since the NSS process began, morethan 175 tonnes of highly enricheduranium (HEU)—enough for nearly7,000 nuclear weapons—has been

removed or down-blended (mostlyfrom Russia).

30 countries have eliminated all HEUfrom their territory, and radiationdetection equipment has beeninstalled at 329 international border

crossings, airports and seaportsto prevent, detect and respond totrafficking in nuclear and otherradioactive material.

  Additionally, the Convention onthe Physical Protection of NuclearMaterials (CPPNM), with the 2005amendment, is now only eightsignatures shy of entering into forceand the International Conventionon the Suppression of Acts ofNuclear Terrorism (ICSANT) hasbeen signed by 103 of the 193 UnitedNations members.

Challenges Remaining 

  An estimated 1,400 tonnes ofHEU and nearly 500 tonnes ofplutonium—enough for about200,000 simple ssion-type nuclearbombs—is still held by more than30 countries.

Moreover, the absence of Russian

President Vladimir Putin (overstrategic differences with the US)indicates that progress towards thiscause is susceptible to the overallstate of bilateral relations.

  The failure to invite Iran (despitethe nuclear deal) was a missedopportunity to engage Tehran on acrucial issue of global importance.

Despite the NSS efforts, thepossibility of terrorist attacks on

nuclear facilities has not beeneliminated even in nations suchas Belgium.

  Countries that in 2010 wereproducing plutonium and highlyenriched uranium continue to do so.

  Both the United States and Russiahave launched massive long-termnuclear weapons “modernisation”programmes.

 THE FOURTH NUCLEAR

SECURITY SUMMIT 

U.S. hosted the rst nuclear security summit in 2010, followed by Seoul in 2012, the Hague in 2014 and Washington D.C. in 2016.

SIGNIFICANCE

This was the fourth in a series of summits that have brought togetherleaders from 50+ countries and four international bodies to make newcommitments towards reducing the threat of nuclear terrorism.

THREAT OF NUCLEAR TERRORISM

  One of the greatest threats to world security.

If any terrorist networks get their hands on a nuclear device, theconsequences for the world would be catastrophic.

BASIC WAYS TO PREVENT THREAT

Locking down nuclear materials and strengthening global nuclearsecurity regime.

KEY TAKEAWAY

The summits pushed for minimising civilian use of highly enricheduranium (HEU), key component used in nuclear weapons, by eliminatingthe material.

Most Important News Events

Page 27: CA June 2016 Eng Xaam.in

8/16/2019 CA June 2016 Eng Xaam.in

http://slidepdf.com/reader/full/ca-june-2016-eng-xaamin 27/68

Current Affairs

72 || Drishti Current Afffairs Today || June 2016

THIRTEENTH INDIA-EU SUMMIT

The Thirteenth India-EU Summitconcluded in Brussels on 30th March2016 without a consensus on a bilateral

free trade deal known as the BTIA(Broadbased Trade and InvestmentAgreement)  even as progress wasmade in bilateral cooperation in otherelds ranging from foreign policy toouter space. The EU and India agreedto continue discussions on a possibleFTA at a high-level.

Key Points

  India has been pushing for openingEuropean markets for its services

sector and the movement of peopleto deliver those services while theEU has been keen on reducing orabolishing tariffs in several sectors,especially in the automobile andwine and spirits sectors.

  The most important thing achievedfrom the Summit was that it putIndia-EU strategic partnership backon track through an expression ofstrong political commitment from

both sides.  The fields of cooperation are

dened by the EU-India Agenda forAction-2020, which was endorsed bythe Indian Prime Minister and thepresidents of the European Counciland European Commission.

  The sectors of partnership rangefrom foreign policy, counterterrorism and disarmament totransport and space.

  There was some promise of action

that would be taken in areas suchas water, climate and energy, withthe adoption of joint declarationson the India-EU Water Partnershipand a Clean Energy and ClimatePartnership.

  India is no longer eligible fordevelopment assistance from theEU. However, India will still haveaccess to concessional loans from theEuropean Investment Bank (EIB).

In the ‘Clean India’ initiative andthe ‘Ganga Rejuvenation’ Initiative,

the EU will help in developing a

solution to clean up the river as wellas developing legal and governanceframeworks for managing the basin.

  Both sides agreed to cooperate in

countering violent extremism,

disrupt recruitment of terrorists

and prevent the free passage offoreign ghters in a joint declaration

on counter terrorism, which alsocalled for the early adoption of theComprehensive Convention on

International Terrorism in the UN. The Common Agenda on Migration

and Mobility (CAMM) designed tocontrol and organize migration wasalso adopted.

Regarding ‘sensitive issues’ thatwere discussed at the summit, bothparties have ofcially expressed theircondence in the legal processes of

the Permanent Court of Arbitration,where the case of the Italian marines,

Massimiliano Latorre and SalvatoreGirone, is currently being heard.The EU also expressed need for aswift solution, ‘through due processof law’ in the case of MV SeamanGuard Ohio, where 14 Estoniansand six Britons were arrested in2013 and sentenced in India.

 Why Tariff Protection isanti-free trade?

The objective of tariffs is

protection of an infant industry tillthey can compete with experiencedplayers in that industry. Theproblem is when it leads topermanent infancy where you havecompanies that never grow up.Therefore, tariff measures arecredible only if they are temporaryand when a sector reaches a certainlevel of competitiveness, free tradeshould be encouraged.

Background on Indo-EU FTA

The talks on the FTA had commencedin 2007 and both sides had held 16rounds of negotiations till 2013.

The talks have been stalled as thenegotiators have so far been unableto address the key demands of theEU and India.

  EU wants lower or elimination ofduties on automobiles and wines &spirits while India's main demandsare focused on data security status(crucial for India's informationtechnology sector to do more

business with the EU rms), easiertemporary movement of skilledprofessionals and seamless intra-corporate movement.

  India has also sought agriculturalmarket access in the EU as wellas disciplining of Sanitary andPhyto-sanitary (norms related withplants and animals) and TechnicalBarriers to Trade to ensure that theconcessions in the FTA that wouldbe given by the EU result in effective

market access. India is keen that theFTA outcome should be balanced.

India, EU andhuman rights issue

  One of the prime reasons forstalled talks between the EuropeanUnion (EU) and India had beenthe EU’s concern over humanrights violations in India.

  The Human Rights Watch(HRW) had written a letter to

the EU stating that the Indiangovernment uses harsh laws toundermine democracy and rightsof marginalized communities.

Note:  Human Rights Watch isan independent, internationalorganization that works to upholdhuman dignity  and advance thecause of human rights for all. It isbased out of New York.

Most Important News Events

Page 28: CA June 2016 Eng Xaam.in

8/16/2019 CA June 2016 Eng Xaam.in

http://slidepdf.com/reader/full/ca-june-2016-eng-xaamin 28/68

Current Affairs

Drishti Current Afffairs Today || June 2016 || 73

PANAMA PAPERS – BIGGEST LEAK OF INSIDE INFORMATION IN HISTORY

The worldwide Panama Papersscandal claimed a fresh political victim

when Spain’s industry minister Jose

Manuel Soria had to resign overallegations of having links to offshorecompanies. Earlier, Iceland’s Prime

Minister  Sigmundur David Gunn-

laugsson was also forced to resignover the leaks. Police also raided theheadquarters of the Panamanian law

 rm Mossack Fonseca whose leakedPanama Papers revealed how theworld’s wealthy and powerful usedoffshore companies and tax havens to

stash unaccounted assets.

The Panama Papers

The ‘Panama papers’ are a set ofover 11 million leaked  documents that reveal how the rich and connected

around the world used tax havens for

storing away their unaccountedwealth.

The leaked documents refer to theclients of Mossack Fonseca, a law

firm headquartered in Panamaand regarded as one of the world’sbiggest creators of shell companies.

These documents were leaked to aGerman newspaper (Suddeutsche

Zeitung),  which in turn sharedit with the Washington basedInternational Consortium of

Investigative Journalists (ICIJ).

  The ICIJ is an international coalitionof more than 100 media outlets.

 What are Shell Companies?

  A shell company is a company without active business operations or

signicant assets.

Shell companies are not necessarily illegal or illegitimate, as they often

serve an important role for potential startups. Additionally, shell companies

can act as a tax avoidance technique for legitimate businesses.

  People or companies might use them to reduce their tax bill legally, by

beneting from low tax rates in countries like Panama, the Cayman

Islands and Bermuda which are referred to as Tax Havens. 

The practice is frowned upon, particularly when used by politicians, who

then face criticism for not contributing to their own countries’ economies.

  The offshore accounts and companies also hide the names of the ultimateowners of investments so they can be used to illegally evade taxes or

launder money.

How are offshore accounts used to evade tax obligations?

What are offshore accounts?

Offshore bank accounts and other nancial dealings in another country

can be used to evade regulatory oversight or tax obligations. Often, companies

or individuals use shell companies, initially incorporated without signicant

assets or operations, to disguise ownership or other information about thefunds involved.

What are the legitimate uses of offshore accounts?

Companies or trusts can be set up in offshore locations for legitimate

uses such as business nance, mergers and acquisitions and estate or tax

planning.

What are the illicit uses of such accounts?

Shell companies and other entities can be misused by terrorists and

others involved in international and nancial crimes to conceal sources offunds and ownership.

Crack down on nancial havens

The Financial Action Task Force and other regulatory agencies publish

assessments identifying weaknesses in enforcement of anti-money laundering

and counter-terrorism nancing efforts of specic countries and territories.

Financial and legal professionals are given training on how to spot potential

violations, since in some cases lawyers and bankers are themselves unaware

that they are handling illicit transactions.

Most Important News Events

Page 29: CA June 2016 Eng Xaam.in

8/16/2019 CA June 2016 Eng Xaam.in

http://slidepdf.com/reader/full/ca-june-2016-eng-xaamin 29/68

86 || Drishti Current Afffairs Today || June 2016

2Constitutional & AdministrativeUpdates

SUPREME COURT /HIGH

COURTS VERDICTS

SC nod to common medical

entrance test NEET

On 28th April, the Supreme Courtapproved conducting NationalEligibility Entrance Test (NEET) forstudents seeking to enter BDS andMBBS courses in the 2016-17 session.

The Supreme Court declined thesubmission of certain state govts. andprivate medical colleges not to thrustNEET upon them.

The court approved the CBSEschedule to conduct the exam on May

1, 2016 as phase-1 of NEET,followedby second phase on July 24, 2016.

Earlier also the Supreme Courthas asked for the implementation ofNational Eligibility Entrance Test(NEET) reversing its own June 2013order as per which the commonentrance test for admissions to MBBS,BDS and PG courses in all medicalcolleges was cancelled. This hasrevived the idea of holding a nationaltest to ascertain the aptitude andsuitability of those seeking to studymedicine anywhere in the country.Over 90 medical entrance tests are held

across India putting the students andtheir parents under lot of difculty.

Background 

  NEET was introduced in 2010through amendments to existingregulations relating to medical anddental admissions.

  The main aim was saving studentsthe trouble of writing multipleentrance examinations to medicalcourses in State-run and privateinstitutions.

  C u r b i n g t h e i n c r e a s i n gcommercialisation of highereducation in medicine.

  Ensuring a transparent admissionprocess in private, unaidedinstitutions which thrive on selling

MBBS and postgraduate medicalspecialty seats to the highest bidder.

Opposition to NEET 

  State governments were upset with

the implicit centralisation of medicaleducation in the idea of a nationaltest as they feared that NEET would

undermine their reservation policy.

  Private institutions, especially those

established by minorities, wereagainst any interference in theiradmission process, arguing that their

unfettered right to regulate theirown admissions had been upheld by

an 11-judge Supreme Court Bench

in T.M.A. Pai Foundation (2002).

Note:  In the T.M.A. PaiFoundation v. State of Karnataka,(2002) case the Supreme Court heldthat the unit to determine a religiousor linguistic minority can only bethe State. Thus, religious andlinguistic minorities, who have beenput on a par in Article 30, have tobe considered State-wise.

SC moves IPL Matchesout of Maharashtra

Supreme Court dismissedMumbai Cricket Association’s pleaagainst Bombay High Court order onshifting the IPL matches out ofMaharashtra.The cricket associationin its plea had said that it will not beusing potable water for the cricketpitches but use treated sewage waterinstead.

The Bombay High Court had onApril 13 ordered shifting of all IPLmatches scheduled in Maharashtraafter April 30 to another state in viewof the severe drought in the state.

The court said that in view of adrought situation in the State wherethere is not a drop of water availableand all dams are drying, how the Statecan turn a blind eye to the scarcity ofwater.

Wasting water on IPL matches

BCCI had planned twentyT20 matches in water-starvedMaharashtra and just for the sevenIPL matches at the Wankhedestadium approximately 40 lakh litresof water would have been used.

  The court came down heavily on theBCCI terming the use of water tomaintain cricket pitches as “criminalwastage” when the state is facing

a drought-like situation.  The court is correct to raise the

moral question: Is it right to goahead with cricket matches whenwe really have a water crisis of hugeproportions on hand?

 Is shifting the IPL matchesa right decision?

  The shifting of matches will donothing to solve Maharashtra’swater problems, as the HC Bench

itself admitted.

  The amount of water used tomaintain cricket grounds is aninsignicant fraction of the State’swater consumption.

On the other hand Bombay HCorder has succeeded in drawingattention to the seriousness of thedrought situation and the grossinequities that prevail in the waypeople access water.

Page 30: CA June 2016 Eng Xaam.in

8/16/2019 CA June 2016 Eng Xaam.in

http://slidepdf.com/reader/full/ca-june-2016-eng-xaamin 30/68

Current Affairs

Drishti Current Afffairs Today || June 2016 || 87

  One may argue that life will not stopfor the crisis and that the water saved

will not make any difference in theoverall situation of drought but theprinciple of moral imperative mustnot be overlooked here.

  Cricket is a game loved by themany Indians, but it exists in socio-economic milieu from which itcannot be divorced.

Seriousness of WaterCrisis in Maharashtra

  In Marathwada , dams and reservoirsare running dry and tankers aredoing booming business.

Section 144 has had to be imposedto prevent a water riot in Laturwhere the traditional sources ofwater have run dry.

Current data from the CentralWater Commission shows thatwater level in 91 major reservoirsis alarmingly low.

  Groundwater is getting over-

exploited, especially in the GreenRevolution zones.

Steps needed to mitigate watercrisis

  There have been two years ofdecient rainfall and the water-splurging agri-economy needsurgent policy intervention.

  Reworking of the price supportregime for crops and rationalisation

of electricity subsidies are requiredto shift the farmer towards less

water-hungry crops.  Deepak Pental, former vice

chancellor of Delhi Universityand genetics professor, has pointedout that when India exports 1 kg ofbasmati rice it is in effect exporting5,000 kg of water.

BCCI challenges LodhaCommittee recommendations

The Supreme Court (SC) has been

blamed of crossing judicial limits bythe Board of Control for Cricket inIndia (BCCI) for interfering in itsprivate affairs like internalmanagement, ow of nances earnedfrom distributing media rights formatches and membership patterns.This prompted the court to ask theBCCI whether it was “refusing to bereformed”. Mumbai’s Cricket Club ofIndia (CCI) said that the Justice LodhaCommittee recommendations affected

Article 19 (1) (c), which enshrines thefundamental right of citizens to formunion or associations under theIndian Constitution.  The SCresponded to that argument by statingthat there are no private citizens butonly individual associations on theBoard of BCCI and therefore there isno question of the Lodha Committeerecommendations infringing onanyone’s fundamental rights.

 Lodha committee recommendation

Supreme Court had appointed Justice RM Lodha committee tosuggest reforms in structure andfunctioning of the BCCI in theaftermath of spot-xing scandal in

Indian Premier League (IPL). Thecommittee’s recommendations aim to

resolve issues of governance,transparency, and conict of interest

in BCCI. The committee’s importantrecommendations are:

  Betting in cricket should be legalized

to curb corruption in cricket except

for players and ofcials.

The BCCI should be brought under

the Right to Information Act to share

administrative and nancial detailswith the public.

  No ofcial should hold a position

for more than three tenures of three

years each but there should not betwo consecutive terms.

Maximum age limit for holding apost should be 70 years.

  Ministers and bureaucrats cannotbe BCCI ofce bearers.

Only one association from a stateshall be a full member of BCCI with

voting rights. Other associationsfrom states and boards like Railway

Sports Promotion Board shouldhave only associate membershipwithout voting rights.

  A nominee of the Comptroller andAuditor General should be included

which will keep an eye on howthe board’s nancial resources are

being utilized.  The IPL should have a separate IPL

governing council.

SC can’t be bypassed on

inter-State disputes

The Haryana government hassubmitted in the Supreme Court thatno State Assembly can pass a law tonegate the apex court’s constitutionalpowers to adjudicate and decide

Constituti onal & Adminis trative Updates

Page 31: CA June 2016 Eng Xaam.in

8/16/2019 CA June 2016 Eng Xaam.in

http://slidepdf.com/reader/full/ca-june-2016-eng-xaamin 31/68

Current Affairs

88 || Drishti Current Afffairs Today || June 2016

inter-State disputes by invoking judicial precedents in the Cauveryriver water sharing dispute caseinvolving Tamil Nadu, Kerala andKarnataka. In a hearing of thePresidential Reference on the validity

of the Punjab Termination ofAgreements Act of 2004 — which has

 jeopardised the Sutlej-Yamuna LinkCanal project — before a Constitution

Bench, senior advocate appearing forHaryana stated that a law passed bythe State legislature to circumvent orrender infructuous a Supreme Courtverdict is a clear encroachment by thelawmakers into the judiciary’s terrain.

 Punjab Sutlej-Yamuna Link Canal

(Rehabilitation and Re-vesting of Proprietary Rights) Bill

On March 14, even as the SupremeCourt began hearing the President’sReference on the 2004 Act, the PunjabAssembly went ahead and passed thePunjab Sutlej-Yamuna Link Canal(Rehabilitation and Re-vesting ofProprietary Rights) Bill, whichprovided for the return of over 5,000acres of land acquired from farmers

for the canal.

Responding to the Punjab Bill, the

SC on March 17 ordered maintenance

of status quo and made it clear that it

would not be relegated to the status

of a “silent spectator” by any State

Assembly. The court appointed the

Union Home Secretary, Punjab’s ChiefSecretary and Director General ofPolice as joint receivers to ensure no

alterations were made in the canal

structure and related properties.

What is the Sutlej Yamuna Link

(SYL) Canal controversy?

  The creation of Haryana from the old

(undivided) Punjab in 1966 threwup the problem of giving Haryana

its share of river waters. For Haryana to get its share of the

waters of the Sutlej and its tributary

Beas, a canal linking the Sutlej with

the Yamuna was planned (SYL

Canal).

Available supplies were calculated

to be 17.17 MAF (Million Acre Feet),

and Punjab, Haryana and Rajasthan

were allocated 4.22 MAF, 3.5 MAF

and 8.6 MAF respectively. Jammu

and Kashmir and Delhi got 0.65MAF and 0.20 MAF.

Haryana moved the Supreme Court

in 1996 seeking directions to Punjab

to complete the work on the SYL.

   In 2002, and again in June 2004, the

SC directed Punjab to complete the

work in its territory.

But within a month of the Supreme

Court order, on July 12, 2004, the

Punjab Assembly passed The PunjabTermination of Agreements Act,

2004, terminating its water-sharing

agreements, and thus jeopardising

the construction of SYL in Punjab.

Why has the SYL issue taken

centre-stage again now?

  Supreme Court in March 2016

started hearings into a presidential

reference to decide on the legality

of the Punjab Termination of

Agreements Act, 2004.

The presidential reference was made

after the Punjab Assembly passed

the Act. As the hearings resumed,

the Solicitor General said that the

Centre stood by the SC’s orders

asking Punjab to complete the work

on SYL in its territory.

   The development has triggered a

political storm in Punjab and with

the elections in Punjab due next

year , all parties are keen to gain

political mileage.

 What is Presidential

Reference?According to Article 143 of the

Constitution of India, the President

of India may refer to the Supreme

Court of India, a question of law

or fact which, he thinks, is of public

importance.

The opinion is only advisory,

which the President is free to follow

or not to follow. However, even if

the opinion given in the exercise ofadvisory jurisdiction may not be

binding, it is entitled to great weight.

The Supreme Court may

decline to give its opinion under

Article 143 in cases it does not

consider proper or not amenable

to such exercise. It was, however,

held by the Supreme Court in

M. Ismail Faruqui v. Union of India

(1995)  that in such a case reasonsmust be indicated.

It was also held by the Supreme

Court that the references made

under this Article are not the “law

declared by the Supreme Court”

under Article 141 of the Constitution.

So it is not binding on inferior courts,

even though they have high

persuasive value.

Constituti onal & Adminis trative Updates

Page 32: CA June 2016 Eng Xaam.in

8/16/2019 CA June 2016 Eng Xaam.in

http://slidepdf.com/reader/full/ca-june-2016-eng-xaamin 32/68

96 || Drishti Current Afffairs Today || June 2016

INDIAN ECONOMY

100% FDI in e-commercemarketplace model

The government has allowed100% foreign direct investment (FDI)through the automatic route in themarketplace model of e-commerceretailing making clear the FDI policyfor the sector as well as the denition

of marketplace format. Foreign direct

investment has not been permitted ininventory-based model.

  At present, 100% FDI is permittedin B2B (business-to-business)transactions under the automaticroute.

The marketplace model hasbeen defined as providing an“information technology platformby an e-commerce entity on a digitaland electronic network to act as a

facilitator between buyer and seller.”  The e-commerce marketplace may

provide support services to sellersbut should not exercise ownershipover the inventory because it willrender the business into inventory-based model.

  An e-commerce rm will not bepermitted to sell more than 25%of total sales from one vendor orits group companies.

E-commerce entities providingmarketplace should not directly orindirectly inuence the sale priceof goods or services.

 Is 100% FDI in e-commercemarketplace model a win-win forall?

  The move to allow 100% FDI inmarketplace e-commerce companiesis designed to protect ofine retailers(mom and pop shops).

3  Economic Scenario

  Barring marketplaces from offeringhefty discounts would adverselyaffect the end consumers who havebeen the biggest beneciaries of thediscount-driven business model.

  In a free market economy,policymakers should not be tellingbusinesses what they can charge fora product especially if it is not anessential life-saving commodity.

  The big risk is that such policy may

end up raising the barriers to entryof capital into the sector.

Policymakers need to make surethat policies governing the sectorare designed to facilitate the futureand not regulate the past.

India appeals WTO solar ruling 

India has appealed to the WorldTrade Organization’s highest court—the Appellate Body—to dismiss alower panel ruling that struck downthe India's domestic contentrequirements (DCR) for solar cells andmodules following a complaint by theUS. The dispute settlement panel ofWTO had ruled that India cannotdiscriminate between foreign anddomestic suppliers of components forsolar panels used by solar powerdevelopers in India.

Background 

  The WTO panel had found India’sDCR under the Jawaharlal NehruNational Solar Mission (JNNSM)violative of its commitments underthe General Agreement on Tariffsand Trade (GATT) and TradeRelated Investment Measures(TRIMs).

  The Indian government hadmandated the solar powerdevelopers to use solar cells andmodules manufactured in India

in different phases of JNNSMand agreed to buy the electricityproduced at a xed rate for 25 years.

  The WTO panel agreed with thecontention of U.S. that by imposingDCR, India was according importedsolar cells and modules less favorabletreatment than similar productsmanufactured in India.

India had argued that since thegovernment ultimately procured

the electricity produced, it should betreated as government procurementand thus exempt from WTO rules.But the WTO panel held that theproduct procured was electricity,while the product discriminatedagainst was the inputs used toproduce electricity.

  India also tried to justify the DCRunder the general exceptionsprovision of the GATT claiming that

it was necessary to secure compliancewith its commitments underthe United Nations FrameworkConvention on Climate Change(UNFCCC) and also because solarpanels were in short supply.

  The WTO panel rejected botharguments stating that internationalenvironmental obligations imposedno binding commitments onnations and also found that thecumulative supply of both foreign

and domestic solar cells and moduleswas sufcient to meet the demandof power developers.

Mallya‘s resignation rejected

The liquor baron Vijay Mallya hadresigned from the Rajya Sabha (RS)on 2nd May, a day before the HouseEthics Committee was to recommendhis expulsion. But his resignation hasbeen rejected by the Chairman of RS.

Page 33: CA June 2016 Eng Xaam.in

8/16/2019 CA June 2016 Eng Xaam.in

http://slidepdf.com/reader/full/ca-june-2016-eng-xaamin 33/68

Current Affairs

Drishti Current Afffairs Today || June 2016 || 97

 India writes to U.K.on Mallya's deportation

The Ministry of External Affairshas written to the High Commissionof the United Kingdom in Delhirequesting deportation of Vijay Mallya

from the United Kingdom wanted formoney laundering and nancialmismanagement.

The Ministry of External Affairshad earlier revoked the passportof Vijay Mallya.

A non-bailable warrant issued bySpecial Judge, Mumbai under thePrevention of Money LaunderingAct (PMLA), 2002.

  Mr. Mallya still has to dischargehis liabilities quantied at about

Rs.9,000 crore.

He has challenged the determinationby banks that he is a ‘wilful defaulter’by arguing that he is personally not aborrower, and only gave a personalguarantee for corporate loans.

He also has to answer the CentralBureau of Investigation’s chargethat the Rs.900-crore loan KingsherAirlines had taken from IDBI Bankinvolved money-laundering.

 

In March, a Hyderabad court alsoissued a non-bailable warrantagainst him for non-appearancein a case of the alleged dishonourof a cheque for Rs.50 lakh issuedto GMR Hyderabad InternationalAirport Limited.

 Lessons for banks fromthe Mallya case

  Individual public ofcials should notbe allowed to dispense such large

sums of public money. Move to asystem of vetting loan applicationsthrough committees, thus allowing

 joint responsibility to come into play.

  Lack of transparency weakensour public banking sector. Detailsincluding every loan sanctionedby public sector banks , includingthe history of the borrower, andthe officials involved should beposted on the website of the bankconcerned.

  The fear of retrospective scrutiny bythe Central Vigilance Commissionerhas driven loan ofcers into inaction.It is of no use having a publicly-owned banking system that doesnot extend credit to sound projects.Therefore the need of the hour is towork towards a re-engineering ofprocedures to facilitate legitimatefunctioning of banking sector.

Note: Money laundering is the genericterm used to describe the process bywhich criminals disguise the originalownership and control of the proceedsof criminal conduct by making suchproceeds appear to have been derivedfrom a legitimate source.

LPG for every Indian household

The Pradhan Mantri UjjwalaYojana (PMUY) has been lancuhed byPrime Minister on 1st  May 2016(Labour Day)  with the aim ofproviding ve crore subsidisedLiqueed Petroleum Gas (LPG)connections to women belonging topoor households (Below Poverty Line)in the next three years. The PMUY hasproposed payment in installments forstoves and cylinders to address theone time nancial challenge.

Pradhan MantriUjjwala Yojana 

  A Scheme for Providing Free LPGconnections to Women from BPLHouseholds.

   Rs 8000 crore has been earmarkedfor providing five crore LPGconnections to BPL households.

Scheme provides a financialsupport of Rs 1600 for each LPGconnection to the BPL households.

  Identification of eligible BPLfamil ies wil l be made inconsultation with the StateGovernments and the UTs.

  It would be implemented overthree years FY 2016-17, 2017-18and 2018-19.

It is for rst time in the historyof the country that the Ministryof Petroleum and Natural Gas isimplementing a welfare scheme.

 Hurdles in universalizing LPG coverage

  Ensuring reliable, sustained, last-mile supply would require multiplesteps including a large extension ofdistribution networks, especially

in rural areas, since each ruraldistribution agency typically catersto fewer customers than urbanagencies.

  Ensuring reliable supply is alsolikely to require strengtheningthe rening, bottling and pipelineinfrastructure.

  The scheme should be accompaniedby a focussed public relationscampaign, similar to the national

tuberculosis or Swachh Bharatcampaigns, to build awarenessand create a “demand pull”, notonly for clean cooking but also forgood service.

In the absence of such supportingmeasures, the PMUY runs the riskof failing like the Rajiv GandhiGrameen Vidyutikaran Yojana,which succeeded in extendingphysical electricity infrastructure atgreat cost but has not been able toensure a reliable supply of affordableelectricity to households.

  PMUY targets only BPL households,there is a need to widen the netbecause there are errors in BPL listsand BPL may be a narrow denitionof deprivation and many non-BPLhouseholds may also not be able toafford LPG connections.

Effective monitoring and grievanceredressal systems are equally

important to ensure that problemsin the scheme are highlighted andaddressed early.

Bharti’s payments bankunit gets nal RBI nod

Bharti Airtel's payments bankventure Airtel M Commerce ServicesLtd. has become the rst entity toreceive nal approval from the ReserveBank of India (RBI) to start a payments

Economic Scenario

Page 34: CA June 2016 Eng Xaam.in

8/16/2019 CA June 2016 Eng Xaam.in

http://slidepdf.com/reader/full/ca-june-2016-eng-xaamin 34/68

106 || Drishti Current Afffairs Today || June 2016

ASIA

Heart of Asia conference

The Heart of Asia conference

began in New Delhi on 26th April with

the objective of discussing the situation

of peace and stability in Afghanistan.

The conference seeks to speed up

reconstruction in Afghanistan with a

focus on enhancing investment and

connectivity to the country. Energy,infrastructure and investment deals

to shore up economic growth of

Afghanistan were the key concerns

before the participating countries.

 About Heart of Asia Conference

This conference is a part of the

Istanbul Process established in 2011

which provides a platform to discuss

an agenda of regional cooperation

with Afghanistan at its centre. The 14 member countries engage

in result-oriented cooperation for

a peaceful and stable Afghanistan

and, by extension, a secure and

prosperous region as a whole.

Member countries are Russia, China,

India, Pakistan, Afghanistan, Iran,

Kazakhstan, Kyrgyz Republic,

Tajikistan, Turkmenistan, Saudi

Arabia, UAE, Azerbaijan and

Turkey.

Transition to Democracy

in Myanmar

The swearing-in of Myanmar’srst civilian president in half a century

marks a watershed moment for thenation that has seen decades of ruleby a military junta. The swearing-inof President Htin Kyaw follows the

4  International News Events

landslide victory of democracy iconAung San Suu Kyi’s National Leaguefor Democracy (NLD) in nationalelections held in November 2015.Although Suu Kyi herself isconstitutionally barred from becoming

president because her children haveforeign citizenship, the newgovernment has created a special post

of ‘state adviser’ for her. In fact, SuuKyi has said that she will be above the

president and have de facto executive

authority.

Challenges confronting Myanmar 

  Myanmar’s military junta still retainsa substantial political role.

A quarter of the seats in parliamentare reserved for the army and thelatter will appoint its own nomineesfor defence, home, and borderaffairs ministries.

At the same time, the armedforces would be acutely aware

that the people of Myanmar wanta termination of internationalisolation and parity with the restof the ASEAN by way of economic

development.

  Myanmar’s domestic security

situation for decades has been

plagued by multiple ethnic

insurgencies and only by ensuring

full democratic rights to all Burmese

groups, including the marginalizedRohingyas.

Chronology of

 Transformation

  1962: Military junta took reins

  1990: Elections held but militaryrefused to recognise the NLD’striumph ; instead it placed AungSan Suu Kyi under house arrest

  2010: General Thein Sein, formed a

political party — United Solidarityand Development Party (USDP)

  2011: An armed forces-approved,

reformist government underThein Sein was set up

  2012:  Ms. Suu Kyi and some ofher colleagues granted entry into

Parliament in carefully calibratedby-elections

  2015:  Suu Kyi's NLD win alandslide November victory in

the rst free and fair electionsin decades

Note:  Junta - A group of militaryofcers who govern a country, oftenafter having seized power in a coup.

 India’s Role

Buddhism, which is all-pervading

in Myanmar, spread from India and

civilisationally the two countries

have much in common.

Page 35: CA June 2016 Eng Xaam.in

8/16/2019 CA June 2016 Eng Xaam.in

http://slidepdf.com/reader/full/ca-june-2016-eng-xaamin 35/68

Current Affairs

Drishti Current Afffairs Today || June 2016 || 107

  To its credit India has maintainedsteady links with Myanmarthroughout its democratic transitionperiod as a result of which Indiais in a good position to leveragehistorical and cultural links formutual development.

Myanmar’s wariness aboutoverdependence on China increasesfurther the scope for New Delhi-Naypyidaw cooperation.

  Bilateral security cooperation againstinsurgencies and connectivityprojects such as the India-Myanmar-Thailand trilateral highway  arecrucial for actualizing India’s ActEast policy and galvanising itsnortheast.

Myanmar offers a sizeable, mutuallybenecial economic opportunity forIndia. A consolidation of commercialties can bring prosperity to India’snorth-east and revitalise Kolkataand other eastern ports.

 The Myanmar Saga 

  The junta kept Myanmar inisolation and economic stagnationwhile refusing to listen tointernational counsel since 1962.Suu Kyi came to prominence in1988, when popular protests werebuilding up. The junta crushedthe protests, killing thousandsof people and placing Suu Kyiunder house arrest in 1989.

  Suu Kyi endured decades ofhouse arrest and harassment bymilitary rulers, did not give upon her non-violent campaign tounseat them. She was awardedthe Nobel Peace Prize in 1991

while under house arrest.

  The junta nally started looseningits grip on power in 2010, allowingelections that were won by amilitary allied party after theNLD boycotted the polls.

  Suu Kyi led NLD to a landslidewin in November, usheringMyanmar 's f i rs t c iv i l iangovernment after 54 years ofdirect and indirect military rule

  Suu Kyi is widely known as India’sfriend, a believer in Gandhianphilosophy and an admirer of

 Jawaharlal Nehru. With her inpower, India-Myanmar relationsshould ourish in the normal course.

Saudi warns of retaliation ifU.S. passes 9/11 Bill

Saudi Arabia has told the Obamaadministration and members ofCongress that it will sell off hundredsof billions of dollars’ worth ofAmerican assets held by the kingdomif Congress passes a bill that wouldallow the Saudi government to be heldresponsible in U.S. courts for any rolein the September 11, 2001, attacks.

  The Obama administration haslobbied Congress to block the bill’spassage and warned senators ofdiplomatic and economic falloutfrom the legislation.

  Saudi ofcials have long deniedthat the kingdom had any role inthe 9/11 plot but suspicions havelingered, partly because of theconclusions of a 2002 congressionalinquiry into the attacks that citedsome evidence that Saudi ofcials

living in the U.S. at the time had ahand in the plot.

U.S.-Saudi alliance facesnew pressures

When U.S. President arrived inRiyadh on 20th April to attend aregional summit of Gulf leaders, hewas welcomed by the local governorand not by King Salman binAbdulaziz Al Saud  himself. Giventhat the monarch personally welcomed

other leaders who arrived for thesummit, this is a strong indicator ofthe deepening rift in the U.S.-Saudialliance.

  In the 70 years since the alliance wasinitiated at the end of World War II,the Saudis have primarily looked tothe United States to help ensure itssecurity in an often unstable region.

Saudis want the U.S. to supportthem, especially in a potential

conict with Iran, their longstandingregional rival.

  The United States on the otherhand has looked to Saudi Arabiaas a source of stability in the WestAsian region, an ally whose oil

reserves have only recently begunto diminish in importance for U.S.interests.

Saudi Arabia‘s Vision 2030

Saudi Arabia announced itsVision 2030 reform plan, a packageof economic and social policiesdesigned to free the kingdom fromdependence on oil exports.

Non-oil revenues are to reach600 billion riyals by 2020 and 1trillion riyals by 2030, from 163.5billion riyals in 2015.

  The share of non-oil exports inGDP is to rise to 50% of GDPfrom 16%. Unemployment amongSaudi nationals is to fall to 7%from 11.6%.

  Financial institutions will beencouraged to allocate up to 20%of their overall funding to smalland medium-sized enterprises

by 2030.  During Mr. Obama’s tenure, there

has been distrust and disagreement

between U.S. and Saudi Arabia over

how to contain Iran, the ght against

the Islamic State, the future of Syria

and clashes in Yemen.

U.S. is a democracy that hasembedded human rights issueseven into foreign policy actionswhereas Saudi Arabia is a closed

society ruled by a conservative,authoritarian family. But economic

and strategic interests had helpedboth countries set aside thesecontradictions for a long time.

 Future of U.S. – Saudi Arabia ties

  All this doesn’t mean US is going to

abandon Riyadh or embrace Tehran.

Both the U.S. and Saudi Arabia still

need each other.

International News Events

Page 36: CA June 2016 Eng Xaam.in

8/16/2019 CA June 2016 Eng Xaam.in

http://slidepdf.com/reader/full/ca-june-2016-eng-xaamin 36/68

Topper’s Interview 

Drishti Current Afffairs Today || June 2016 || 149

Success consists of going from failure to failure without loss of enthusiasm. ~Winston Churchill

Success seems to be largely a matter of hanging on after others have let go. ~William Feather 

Some people succeed because they are destined to, but most people succeed because they are determined to.

Shoot for the moon. Even if you miss, you'll land among the stars. ~Les Brown

Life's problems wouldn't be called "hurdles" if there wasn't a way to get over them.

Aman Mittal at LBSNAA, Mussoorie

Newton acknowledged others behind hissuccess: “If I have seen further than others,

it is by standing upon the shoulders of

giants.” You wish to clear the Civil Service

Examination as soon as possible and you

think you have done everything that the exam

requires, yet you feel God has been unkind

to you. Well, you are not the only one

complaining; it’s just that the seats are fewand God and UPSC get lakhs of requests and

petitions every year. We cannot guarantee

your success (that is mostly in your hands),

but we can definitely provide you with

inspiration and strategy. In this section of the

magazine, we bring you closer to toppers:

those who already are where you aspire to

be. We interview an exam topper in thesepages so that you can learn from his strategy,

tricks, plans, ideas and mistakes; so that you

can stand on their shoulders and see Mussoorie

on the horizon. One day you will be where

they are now.

Page 37: CA June 2016 Eng Xaam.in

8/16/2019 CA June 2016 Eng Xaam.in

http://slidepdf.com/reader/full/ca-june-2016-eng-xaamin 37/68

Topper’s Interview 

150 || Drishti Current Afffairs Today || June 2016

eventually. And once you areacquainted with the general principles

of Polity, Economics etc. then towards

the later part of you preparation you

should start writing answers.

DCAT:  Did you give specialemphasis to some particularsections or equal emphasis on allsections? In your opinion, cancertain sections be skipped?

Aman:  Generally, one should giveequal emphasis but yes, optional and

the essay are important. The syllabus is

so generally dened, it is difcult to

decide what to skip.

DCAT: How much time did you

take to complete your preparationfor all the three stages of theexamination? Did you prepare foreach stage in sequence or all thestages simultaneously?

Aman: It was basically simultaneously

only because there is no specific

preparation for prelims; the CSAT

was there at that time and obviously

if you are preparing for Mains then

you cover 80 per cent of prelims

syllabus also. And so far as interview

was concerned, that does not require

specic preparation. There are some

nal points that need to be polished.

 Aman Mittal, Rank 20 inaugurates our section on ‘Topper’s

 Interview’. Aman is presently at Lal Bahadur Shastri National Academy of Administration (LBSNAA), Mussoorie undergoingtraining for the IAS. He shares with us his strategy, pains and joysof the CS examination process. How does one prepare for the CivilServices Examination (CSE)? How to manage the ups and downs?What are the useful strategies for studies and preparation? Howto prepare well enough to get a top rank like him? Let’s hear it fromhim.

Drishti Current Affairs Today(DCAT):  How did you feel onbeing selected to the administrativeservices at such a good rank?

Aman Mittal: I felt ecstatic. It was a

great feeling being selected to the civil

services.

DCAT:  Were you satisfed withyour level of preparation beforethe exam and were you hopeful ofbeing successful?

Aman: I was expecting a good result

but could not condently say that I

will get such a good rank.

DCAT: How has life at Mussoriebeen so far?

Aman: Till now, it has been good. I

am looking forward to the eld work.

DCAT:  What attracted you to civilservices?

Aman: My biggest inclination to the

civil services was the diversity of the

 job. You can work in multiple proles

and you can do some research which

you can apply along with a lot of

things you’ve done in your academic

background like engineering.

DCAT: Other than your capabilityand hard work, to whom wouldyou like to give credit for yoursuccess?

Aman: I think I have to give credit toa lot of people. It was only 5-10 per

cent of me and a lot of my success is

dedicated to my mother, my father ,

my brother mostly and my friends

who were with me throughout;

specially for the kind of support they

gave me when I was down. Especially

my friend Vipul: he used to be with

me so much and is an aspirant himself

and I am hoping that he gets through.

And most importantly to God; luck isa big factor.

DCAT:  When did you start yourpreparat ion — along withgraduation, immediately after it orsometime later?

Aman: I started my preparation while

I was at IIT- Delhi. I graduated in 2014

and started my preparation in mid-

2013.

DCAT: Considering the extensivesyllabus of General Studies inPreliminary and Main Exami-nation, what strategy did youadopt in preparing for it?

Aman:  I started with selecting my

optional. Electrical engineering was

the natural choice for me. Secondly,

it is important to go through general

books rst, to get an idea about the

civil service – to avoid getting shocked

Selected as IAS in UPSC CSE 2014 Aman Mittal

Topper’s Interview...

Page 38: CA June 2016 Eng Xaam.in

8/16/2019 CA June 2016 Eng Xaam.in

http://slidepdf.com/reader/full/ca-june-2016-eng-xaamin 38/68

P.T. Express

Preliminary examination is a crucial stage of the exam process even if it is ‘preliminary’ and makes

your eligible for CS (Main) examination. It is a ‘screening’ test which means that you will be examined

methodically by the UPSC through an objective test and you will be shown the door if you are not up to

par and don’t meet the cut-off marks for your category.

Candidates do take PT exam casually at times because they are overcondent due to years of preparations

or history of previous selection(s) or due to ignorance (PT is a small affair for some). Don’t take the PT

exam casually unless you like shocks in life. Be very serious about it; there have been cases where those

nally selected were rejected at Prelims. Examiners at UPSC spend a whole lot of time devising ways tomake it competitive year after year. Don’t rest on your laurels; previous selection or previous academic

record of gold medal in BA or PG will not guarantee you a seat in the Main exam. Only hard, smart work

will. So prepare well for PT exam, which should ideally be miles wide in coverage (vast) and inch deep

(neither shallow, nor deeply researched).

P. T. Express is the section where we give you quick questions to brush up your P. T. preparation. The

name ‘Express’ derives from the quickness and ease that you should feel while going through them. In its

various sections, you can go through many topic-based questions and quickly see queries with answers

to reinvigorate your brain. This part of the magazine should be read on a lazy Sunday afternoon with your

favourite cup of beverage. The idea is to keep you alert without tiring and taxing your mind. We don’twant to ood your mind with UPSC

CSEesque PT questions; but at the same

time, we want you remain alive to the

need of knowing facts, information from

current events or History that the UPSC

is hunting day in day out.

Go ahead, take a look and see where

the P.T. Express leads you. Wake up and

smell the coffee, if you are not doing verygood. Bravo, if you are good at it; keep

studying more so that the P.T. Express

has a seat for you reserved for the CS

Mains. All candidates want to board the

P.T. Express but not every candidate is

willing to do the hard work it takes. The

choice is yours. All the best.

Page 39: CA June 2016 Eng Xaam.in

8/16/2019 CA June 2016 Eng Xaam.in

http://slidepdf.com/reader/full/ca-june-2016-eng-xaamin 39/68

P.T. Express

154 || Drishti Current Afffairs Today || June 2016

  Sati was made illegal in 1829 but was not made punishable

by the courts until– 1840

  Ram Mohan Roy was given the title ‘Raja’ by–

 Akbar Shah II 

  ‘Unhappy India’ was written by–

 Lala Lajpat Rai

  Who drafted the Poorna Swaraj pledge?

 Jawaharlal Nehru

  In Quit India Movement, many smaller zamindarsparticipated in the movement whereas big zamindarsmaintained a stance of neutrality and refused to assistthe British in crushing the movement – True or False?

True  The decision to launch individual Satyagraha Movement

was taken in which Congress session?

 Ramgarh

  In which year the Council of Muslim League acceptedthe Cabinet Mission Plan? 1946

  Who were the ofcial Congress negotiators with CrippsMission?

 Pandit Nehru and Maulana Azad 

  In the Interim Government of 1946, Dr. Rajendra Prasad

held which portfolio?

 Food and Agriculture

  Which agreement was signed between India and Pakistan

in 1950 to resolve the issue of protection of minorities?

 Nehru-Liaquat Pact 

  In which year the Constituent Assembly of India waselected by the Provincial Assemblies? 1946

  Who drafted the Constitution of Muslim League, ‘TheGreen Book’?  Maulana Mohammed Ali

  Who wrote ‘Bandi Jiwan’? Sachin Sanyal

  The Revolt of 1857 at Lucknow was led by–

Begum Hazarat Mahal

  Radhakrishnan Dandsena is associated with whichuprising? Savara Rebellion

  Who started the Bombay Mills Lands Association?

 NM Lokhande

  Who organized the Paharia Revolt? Tilka Manjhi

  Which British commander was defeated by the Santhalsin 1855?  Major Burrough

Indian History

  Who founded the ‘Jat-Pat Torak Mandal’ in 1922, forbreaking caste barriers among the Hindus?

Sant Ram

  Who founded the ‘Deccan Educational Society’?

Vishnushastri Chiplunkar 

  The Royal Asiatic Society was founded by–

Sir William Jones

  Who wrote the book ‘Golden Threshold’?

Sarojini Naidu

  Who are the main architects of the Indian NationalConference?

Surendranath Banerjee and Anand Mohan Bose

  Tilak was opposed to the signing of the Lucknow Pact.True or False?  False

  George Yule became the rst English President of theCongress in 1888 during which session?

 Allahabad Session

  MC Setalwad was a distinguished member of whichorganization?  Madras Labour Union

  Who set up the United India House in the USA?

Taraknath Das and GD Kumar 

  Who said that ‘Congress Movement was neither inspiredby the people nor devised by them’?

 Lala Lajpat Rai

  During India’s freedom struggle which incident led tothat rst All India Hartal?

 Arrival of the Simon Commission

  Who was the viceroy of India during second RoundTable Conference?  Ramsay MacDonald 

  What was the main programme of the Swaraj Party?Council entry

  Why did the Indian nationalist leaders decide tosupport the war effort (First World War) of the BritishGovernment in the beginning?

  In the hope that Britain would repay India'sloyalty by taking it further on the road to self-

 government.

  The rst Satyagrahi selected by Mahatma Gandhi tolaunch the individual Satyagraha in October 1940was– Vinoba Bhave

  Which Governor-General had abolished slavery? Lord Ellenborough

Page 40: CA June 2016 Eng Xaam.in

8/16/2019 CA June 2016 Eng Xaam.in

http://slidepdf.com/reader/full/ca-june-2016-eng-xaamin 40/68

Drishti Current Afffairs Today || June 2016 || 161

Mapping MAP – 1

The Levant is an approximate historical geographical term referring to a large area in the eastern Mediterranean.In its widest historical sense, the Levant included all of the eastern Mediterranean with its islands, that is, it includedall of the countries along the eastern Mediterranean shores, extending from Greece to Cyrenaica. The term Levant

entered English in the late 15th century from French. It derives from the Italian Levante, meaning "rising", implyingthe rising of the sun in the east. As such, it is broadly equivalent to the Arabic term Mashriq, 'the land where the sunrises'. The western counterpart in Arabic is the Maghreb, and Ponente in Italian, meaning 'west, where the sun sets'.

Why in News?

Levant has been in news because in spite of the existing territorial demarcations in the region, the Islamic Stateconsiders it as a unied territory under it. The name ISIL implies Islamic state of Levant(including Iraq and Syria).

Map based questions of Levant region

  1. Identify the water body marked as 1. 2. Identify the important port marked as 3.

  3. Identify the landstrips marked as 2, 6, 10, 5. 4. Identify the countries marked as 7, 8, 9, 11, 12, 13

Page 41: CA June 2016 Eng Xaam.in

8/16/2019 CA June 2016 Eng Xaam.in

http://slidepdf.com/reader/full/ca-june-2016-eng-xaamin 41/68

Drishti Current Afffairs Today || June 2016 || 163

TO THE POINTQUICK, EXAM-READY MATERIAL

1. Gravitational Waves : The Mystery Resolved

 Designed to hit the bull’s eye : point wise, short, crisp material in an easy-to-understand format.

 What are Gravitational Waves?

  They are small ripples or distortions in the curvature of space-time thatpropagate as waves.

 

They are generated in certaingravitational interactions  likeexplosion of giant stars, fusion of theblack holes etc due to the warpingof their surroundings.

  They travel at the speed of thelight, experience no barriers andtransport energy with them, calledas Gravitational Radiation.

  Predicted by Albert Einstein,  onthe basis of his Theory of GeneralRelativity.

 Why in news?

  Last month, India and the US signedan agreement for a new LIGOproject  in India, during PrimeMinister Narendra Modi's visit toWashington (USA).

  Earlier, the gravitational waves werediscovered by the US-based LIGO(Laser Inferno meter GravitationalWave Observatory) in February2016.

  India also participated in the LIGOproject through IndIGO consortium.Since 2009, it contributed around 37scientists from various institutionsof the country like – CMI Chennai,ICTS –TIFR Bengaluru,  IISERKolkata, IISER – Trivandrum, IITGandhingar, IUCAA Pune, RRCATIndore and TIFR Mumbai.

  On March 31, 2016 the US NationalScience Foundation (NSF)  and

India's Department of Atomic Energy(DAE) and Department of Scienceand Technology (DST)  signed aMemorandum of Understanding(MoU) for establishing an advancedgravitational-wave detector in India.

Findings of the Experiment

  The highly elusive  gravitationalwaves were detected for the rsttime.

  It validated the Einstein’s generaltheory of relativity (GTR), accordingto which massive objects caused adistortion in the continuum ofspace–time.

Two LIGO installations, nearly3000km apart, in USA detected thegravitational waves due to mergerof two black holes.

 What is LIGO?

  L I G O ( L a s e r I n f e r o m e t e rGravitational-wave Observatory)is a large scale physics experimentaiming to detect the GravitationalWaves.

  Two observatories, one at Louisianaand the other at Washington, have

been set up to detect the waves withaccuracy and without terrestrialdisturbances.

  Funded by The National ScienceFoundation of the USA government.

The India Connection

  India has been an active partnerthrough IndIGO (Indian initiativesfor the LIGO project), a memberof LIGO scientic collaboration.

  IndIGO is a consortium of advanceexperimental facilities.

  Since 2009, LIGO in collaborationwith IndIGO, has been planninga roadmap towards setting up aGravitational Wave Observatory in

the Asia Pacic region (preferablyin India).

  Almost 37 Indian scientists werethe part of the 1000 member LIGOexperiment team.

  Recently the LIGO-India project,a planned advanced gravitational-wave detector, has got the governmentapproval to be located in India

  It is to be built and operated incollaboration with the LIGO USAand its international partners. In

India, the project will be piloted byDept of Atomic Energy (DAE) andDept. of Science and Technology(DST).

Importance of this Experiment

  It will be a paradigm shift in theway we understand our universe.

  The Newtonian gravity model willgive way to the Einstein’s gravitymodel.

  Discovery of gravitational waveswill help us unlock the secrets aboutevolution of the early universe.

  It will help in receiving the cosmicsignals, earlier completely hidden,thereby giving us more insightsinto the cosmic phenomena in ouruniverse.

  It will help track the variouscosmic phenomena like the blackhole, supernova etc. that shape ouruniverse.

Page 42: CA June 2016 Eng Xaam.in

8/16/2019 CA June 2016 Eng Xaam.in

http://slidepdf.com/reader/full/ca-june-2016-eng-xaamin 42/68

To the Point

164 || Drishti Current Afffairs Today || June 2016

Introduction

  Section 124(A) of Indian Penal Code,1860 deals with sedition.

  One of the most controversial lawsin India. A legacy of British Raj.

  Was used to crush the IndianNational Movement and to dealwith the dissent and disaffectionagainst the government.

  Tilak, Gandhi, Maulana Azadand other great leaders werecharged with sedition by the Britishgovernment.

 

Currently the JNUSU presidentKanhaiya Kumar and some otherstudents were charged with seditionafter the event of anti India sloganson Feb 09.

  Social activist Dr. Binayak Sen and Delhi University ProfessorSAR Geelani have also faced thesimilar charges.

 What the Law says

Whoever, bywords,

 either spokenor written, or by signs or by visiblerepresentation, or otherwise, bringsor attempts to bring into hatred  orcontempt, or excites or attempts toexcite disaffection towards theGovernment established by law, willbe punishable.

Punishment

  Imprisonment for life and/or ne.

 

Imprisonment for3 years and/ 

or ne.

In Post independence India

  Section 124(A) very often clasheswith Freedom of speech andexpression, a Fundamental Rightunder Article 19(1) (a) of the IndianConstitution.

  This provision is frequentlyused against political opponents,

2. Law of Sedition in India

activists and to stie dissent by thegovernments in power.

  Famous cases are Dr. Binayak Sen,

Cartoonist Aseem Trivedi, authorArundhati Roy, JNU studentsKanhaiya Kumar, Umar Khalid etc.

 What the Supreme Court Says

  In Kedar Nath Singh vs State ofBihar (1962),  the constitutionalityof Article 124(A) was challenged infront of the Supreme Court.

  Supreme Court ruled that the clausewas constitutional.

  But it preferred to follow the moreliberal interpretation of the term“sedition”.

  Restricted it only to activitiesinvolving “incitement to violenceor intention or tendency to createpublic disorder or causedisturbanceof public peace”.

  In Balwant Singh vs State OfPunjab (1995) the apex courtobserved that just raising of theslogans,

once or twice by twoindividuals alone cannot be said tobe aimed at exciting  or attemptingto excite hatred or disaffectiontowards the Government.

View of 42nd Reportof the Law Commission

  It favored amendments to Section124(A).

It wanted the scope of actions that

would be punishable under theclause to be widened.

  It proposed the punishment to bexed at a maximum seven yearsand/or a ne.

  At present, a person convictedunder the section can be sentencedto a prison term, up to three yearsor for life — nothing in between!

However, nothing was done toimplement those recommendations.

View of Legal Experts

  Free speech,   like any other

fundamental right, was not

absolute  and could be restricted

on certain grounds like public

order, defamation and contempt ofcourt. "One of these grounds was

also sedition which was removed

from the permissible ground of

restriction from Article 19(2) of theConstitution. But sedition remains

under Article 124 (A), left wholly

to the interpretation of the court".

–Fali Nariman  Considering the validity of the

Section 124(A), in the Kedarnath

case, the section is limited to acts

or words or deeds, which have a

tendency to disrupt public order,

or which incite violence. 

– Soli Sorabjee

  Sedition is a very harsh  Section.

Mere speech doesn’t constitute

sedition. Speech should be

followed by action. –Abhishek Manu Singhvi

Conclusion

  The application of section 124(A) is

subject to article 19 (2) i.e. a person

may be prosecuted for sedition

when his/her actions either affect

the security of state or incite the

people for actual violence against

the state.

Mere speech doesn’t constitute

sedition. Speech should be followed

by action.

  It is the duty of the state to protect

its fellow citizen’s right to speak

though one may not agree with

them.

  Most problems will be solved if

one understands that people are

not wrong but different.

Page 43: CA June 2016 Eng Xaam.in

8/16/2019 CA June 2016 Eng Xaam.in

http://slidepdf.com/reader/full/ca-june-2016-eng-xaamin 43/68

To the Point

Drishti Current Afffairs Today || June 2016 || 165

Introduction

   Public sector banks (PSBs) are thebanks whose over 50% share is

owned by the Government.  There are 27 PSBs which form the

backbone of Indian banking sector.

  PSBs had over 70 per cent marketshare both in credits and deposits,as of Dec. 2015. ( Source: RBI)

  The spiral ing NPAs   (Nonperforming Assets) in PSBs since2009 have reached an alarminglevel in 2015-16.

  With almost Rs. 4.5 lakh crores of

bad debts, PSU banks are facing acrisis like never before.

  Stressed assets (NPAs plus loansthat have been restructured) for thePSBs have steadily increased overthe last ve years and are estimatedto reach over 14% per cent of thetotal advances.

Reasons Behind the Problem

  Aggressive lending   by banks(mostly PSBs) during good

times(2004-08)  Sluggishness in the domestic

growth during the recent past.

  Slowdown in recovery in the globaleconomy as well as uncertainty inthe global markets.

  External factors  including theban in mining projects, delay inenvironmental clearances affectingpower, iron & steel sector.

  Nexus of corporate, bankers andbrokers where huge loans weregranted without due diligence.(Vijay Mallya’s case)

  Political interference in the PSBsbusiness related decision making.

Regulated lending like PrioritySector Lending, which sometimesclashes with sound businessprinciples.

  Tendency of Evergreening of loans(Debt Restructuring) rather thanRecovery.

 Why this is a Big Concern

  Growing NPAs has choked the

funding capacity of PSBs which willfurther hamper the credit growthand subsequently the economicgrowth.

  This dangerous phenomenon iscalled as the Twin Balance SheetSyndrome  in Economic survey2015-16 where Growing NPAs

are stressing PSBs balance sheet,

while the choked funding is further

stressing the corporate balance

sheet which is already suffering

due to sluggish global demand.  High NPAs will further reduce the

investors’ condence in the PSBswhich will hamper their fund raisingpotentials for business expansionand growth.

  A vicious cycle will be created wherethe macroeconomic stability willbe hampered by the weaker PSBs.

  It indicates   the operational

shortcomings of the PSBs and 

raises a big question over theirfunctioning, as the Private SectorBanks are still able to manage theirNPAs within almost the safe limits.

Measures taken by

the Government

  Finance ministry has proposed toset up an Asset ReconstructionCompany (ARC) for the stressedsectors like Power and Road.

  Recapitalization drives to enhancePSBs capital assets.

  Setting up of Bank Board Bureau

to deal with management of PSBsand issues such as raising capital.

  Proposal of bank merger to createstronger banks.

  Bankruptcy Code Bill for easy exitof rms and for easy recovery ofstressed assets.

3. Bad Debt Crisis in Public Sector Banks

  Allowed hiring of talents from

private sector for more professionaland sound business practices.

The Way Out

  The Economic Survey 2015-16

has suggested 4R’s as  a rescueprogramme:

  Recognition-  of true value of

assets.

  Recapitalization-  by Govt. to

improve PSBs capital position.

  Resolution- selling/rehabilitating

stressed assets.

  Reform-   work on mission 

‘Indradhanush’ to revamp PSBs

  Simplest way is recovery of NPAsthrough SARFAESI or Debt

Recovery Tribunals.

  Lok Adalats can provide a speedyand mutually acceptable solution.

How to Prevent

it in the Future

  Conservative lending  by PSBs (afterproper due diligence) to the NPAprone sectors like infrastructure,steel, power etc.

  Infrastructure projects to be funded

through alternate routes likeInfrastructure Debt Funds orInfrastructure Bonds etc.

Credit appraisal process shouldbe improved.

  Proper monitoring of warning

signals and corrective measures bythe banks before the asset becomesstressed.

  Recover rather Restructure. 

Name and shame the   wilfuldefaulters.

  Diversifying   and extending theconsumer base of the PSBs to counterthe vulnerability of slowdown incertain sectors.

Page 44: CA June 2016 Eng Xaam.in

8/16/2019 CA June 2016 Eng Xaam.in

http://slidepdf.com/reader/full/ca-june-2016-eng-xaamin 44/68

To the Point

166 || Drishti Current Afffairs Today || June 2016

Introduction

  Global Brent crude oil prices havetumbled down to its lowest levelin January 2016 in last 10 years atUSD 28/ barrel.

  Currently, it is at one third level

of the Jan 2014 prices of US$ 120/barrel.

  Crude oil, forming the largest

baskets for the India’s import

bill, has a lot of signicance for the

Indian Economy.

Reasons for Decline in Price

  Prices have fallen because supplies

have outrun demand.

  US   with enhanced shale oil

production has become the world’snew “swing producer” of oil.

  OPEC is unable to work as a cartel

to set the price of oil by controllingthe production.

  Instead of cutting production todefend prices, OPEC countries

decided to defend market share to support their economies.

  Extra production especially by

Iran, Iraq and Libya.

  Global slowdown, particularly inChina one of the biggest consumerand the importer is further puttingdownward pressure on the globaldemand of oil.

Positive Implications

for India

  India, importing over 80% of its totaloil requirements with over 1/3rdof import bill comprised of crudeoil, is a net beneciary.

  Current account defcit is narrowing

and Foreign Exchange reserve isexpanding.

Cheaper oil may fuel the industrial

growth.

  Ination is expected to reduce, as

cheaper oil will bring down the

cost of transportation, energy and

industrial goods.  It will reduce the Fiscal Decit as

the subsidies on oil products will

come down signicantly.

  Rupee will be strengthened as less

pressure on the Rupee to buy US$

for the oil imports.

Negative Implications

for India

  A large part of external remittancescomes from the oil rich Gulf, West

Asia and North Africa whose

economies are bleeding due to sharp

decline in oil prices.

  This will have huge implications on

India’s Balance of Payment (BOP),

a large part of which is nanced by

these remittances.

  Cheaper oil may shift focus away

from green technology options.

  Indian companies have substantial

investments, trading and nancial

interests in Venezuela, Russia,

Nigeria and Gulf countries which

may suffer adversely with crashed

oil prices.

  Another big shock will come on the

front of Indian migrant workers

working in oil dependent economies

where over half of the India’s 15

million strong migrant workerswork.

  Crash in oil prices have led these

economies to spending cuts and

slashing employments, even

retrenchment of migrant workers.

The Case of Kerala

  Kerala led the decade of migration in

India; it topped both the external and

4. Decline in Global Crude Oil Prices : Boon or Bane for India?

internal migration and contributes

to over 25% of the total external

remittances.

  It has 2.4 million external migrantssending over 1 lakh crore and also

hosts 3 million strong domestic

migrants from Bihar, West Bengal,

Odisha etc. sending over 20,000

crore back to their respective states.

  But now with the economic crisis in

the oil rich countries and returning

of Indian migrant workers, stress

will be further felt on the domestic

migrants to lose their job in Kerala.  If reverse migration starts, it will

hamper the larger socio-economic

goals of poverty reduction and

enhancements in life standards.

The Way Ahead

According to the Economic Survey

2014-15, the decline in oil prices is

an opportunity to rationalize the

energy prices by getting rid of the

distorting subsidies whilst shifting

taxes towards carbon based fuels.

  With increasing Excise duty  on

petrol and diesel as an implicit

carbon tax, India is shifting from a

carbon subsidy regime to a carbon

taxation regime.

  India’s crude oil import bill is likely

to dip by 35% from $120bn in 2014-

15 to $73bn in 2015-16.

With oil dependent economiesfacing fund crisis, India may go for

acquisition of oil rms and blocks

abroad, with this saved import bill.

  A part of this saving may be used

for R&D for green energy options.

  This is a huge opportunity to build

up strategic reserve  of crude oil

at these rock bottom prices for the

purpose of energy security.

Page 45: CA June 2016 Eng Xaam.in

8/16/2019 CA June 2016 Eng Xaam.in

http://slidepdf.com/reader/full/ca-june-2016-eng-xaamin 45/68

To the Point

Drishti Current Afffairs Today || June 2016 || 167

 What is Net Neutrality?

  The term “Net Neutrality” wascoined by the Columbia Law Schoolprofessor, Tim Wu.

  It means treating all data packets at

parity, irrespective of their content,source or destination.

  It prohibits blocking  or slowingdown access to any website orcontent on the web. It also prohibitsproviding competitive advantage tospecic individual apps or servicesdepending on either quality ofservice or pricing.

Guiding Principles

of Net Neutrality

  Choice and control by users overtheir online activities.

  Transparency of data ow decisions

for fair and impartial access tointernet resources.

  Reasonable network managementpractices, neither anticompetitive

nor prejudicial.  Regulatory monitoring   to take

care of quality degradation and tomaintain high standards.

 Why in News?

  Recently it has been noticed thatsome telecom companies areunderstood to be working on a"bypass solution" against the TRAIguidelines on net neutrality, through

virtual private networks or intranet.  Earlier, in February 2016, TRAI had

ruled in favour of Net Neutrality

and prohibited service providersfor charging differential anddiscriminatory charges for dataservices.

  It scrapped Airtel Zero and Free

Basics of Reliance-Facebook, forviolating Net Neutrality due todifferential pricing.

Different Stakeholdersand Their Views

 

Internet Service providers (ISP): Usually, the telecom serviceproviders are the ISPs also. Theybelieve that differential pricingand services is as per marketprinciple and it will help them to getadditional revenue, which is underpressure due to high spectrum andlicense charges as well as businessloss due to stiff competition fromthe VoIP (Voice over InternetProtocol) services like Skype, Viber,WhatsApp etc.

  Net Neutrality Opponents: Accessis more important than choice.Limited access is better than noaccess. Facebook justied its FreeBasics as an effort to bring morepeople online.

  Net Neutrality supporters: Theyare mainly the NGOs, civil societygroups and Start ups. They believethat allowing access to a few appsand web services for free will be

against the spirit of competition,innovation and freedom of customerchoice.

  TRAI: The telecom regulator isresponsible for the growth oftelecommunications and to providea level playing eld and facilitatefair competition in the market. Itconsulted all the stakeholders andnally ruled in the favour of NetNeutrality.

Pros of Net Neutrality  Prevents  the ISPs from charging

for online services for "fast lanes".

  Avoids discrimination  amongusers ensuring similar access toinformation for people of differentsocio-economic status.

  Helps to promote freedom  ofchoice, as ISPs cannot obstruct orincentivize particular contents orsites over others.

5. Net Neutrality

  Promotes a level playing eld forcompeting companies.

Cons of Net Neutrality

  It is against free market rules, where differential pricing is allowedfor different level of services. (Forexample in railways, airlines,energy etc.)

  With the help of sponsorships,some mobile telecom operators mayoffer free internet access to limitedcontent, enabling those who don’thave data on their smart phonesto access limited content for free.

Example- Free Basics.

Challenges for Net Neutrality

  Blocking and filtering   of thecontent for regulatory controls bythe government or by the Internetservice providers to pursue theirbusiness objectives.

  Giving preferential networktreatment to certain data streamover others by creating internet

fast lanes.  Zero- rated services  have the

capacity to shift the internet trafcin a discriminatory way.

  Throttling of data throughput ratesfor data ow in a controlled andpreferential manner.

 Way Ahead

  Digital India  Programme gives“digital access” a top priority. Thegovernment must balance  the

freedom of choice with the accessdilemma.

  In the name of net neutrality,u n n e c e s s a r y g o v e r n m e n tintervention and forcible leveling ofthe playing eld should be avoided.

  India which is on the cusp of Startupboom, primarily of internet relatedstart ups, needs deeper internetpenetration as well as free and openaccess to the internet.

Page 46: CA June 2016 Eng Xaam.in

8/16/2019 CA June 2016 Eng Xaam.in

http://slidepdf.com/reader/full/ca-june-2016-eng-xaamin 46/68

Drishti Current Afffairs Today || June 2016 || 173

Ques.1: Bombay High Court has saidthat Indian Premier League (IPL)matches should ideally be shifted outof Maharashtra to places where thereis no water crisis. Discuss.Answer: The judgment of the BombayHC to shift matches out of Maharashtracame in response to a PIL (PublicInterest Litigation) led by a NGOopposing the use of 60 lakh litres ofwater to maintain cricket pitches

hosting IPL matches in the state whichis reeling under a severe drought.

The court questioned the rationaleof organizing these matches in a statewhere farmers are committing suicidesdue to failure of crops brought on bythe drought and where even hospitalshave been forced to stop surgeries dueto lack of water.

Despite the Mumbai CricketAssociation insisting that only non-potable water was being used to

maintain pitches, the Court called it a‘criminal wastage’ and questionedwhy more importance should be givento IPL matches than peoples’ lives andlivelihood.

The court criticized the stategovernment for turning a blind eye tothe problems of people. The issue isnot merely shifting the IPL matchesout of the state but more importantlydrawing attention of the authoritiesand civil society to the plight of those

Potential Q&AsBased on Current Affairs

affected by the drought. Through this judgment the court has tried to drawattention to the miserable watersituation in the state as well as thecountry with the hope that it generatesawareness and results in concrete stepstaken by government and the citizenryto tackle the root-causes of the drought.

Ques.2:  What is the controversysurrounding Aligarh MuslimUniversity's 'minority' status? What

safeguards have been provided in theConstitution to minority institutions?Answer: The Centre has decided towithdraw its appeal against theAllahabad High Court judgmentdeclaring AMU not to be a ‘minorityinstitution’ as it was set up by an actof parliament and not by the Muslimcommunity.

The Mohammedan Anglo OrientalCollege was established in 1875 by SirSyed Ahmad Khan. It was made auniversity by the Indian LegislativeCouncil in 1920. The Supreme Court,in its judgment in the Azeez Basha casehad ruled that AMU was not a minorityinstitution since it was set up by theBritish legislature. After a wave ofangry protests by Muslims, theparliament in 1981 amended the 1920act to restore the minority status ofAMU. Post this, any dispute oruncertainty over the minority status

of AMU should have ended. However,in 2005, when certain aspects of theadmission policy of AMU werechallenged, the Allahabad HC ruledthat the 1981 Act was ultra vires of theConstitution and once again withdrewAMU’s minority status. This judgmentwas challenged in the Supreme Courtby the Centre, which now standswithdrawn.

This controversy raises the larger

issue of conict between the judiciaryand the legislature and also thesafeguards given by the Constitutionto minority run institutions. UnderArticle 30(a), all minorities whetherreligious or linguistic, have the rightto establish and administer educationalinstitutions of their choice. Being afundamental right, it is protected bya prohibition against its violationunder Article 13 and hence should begiven the widest amplitude and not

read in narrow and literal terms aslong as the institution has been formedwith the intent of beneting theminority community.

Ques.3:  The agreement signedbetween the European Union andTurkey is intended to seal illegalmigration but has faced criticismfrom human right groups and aidagencies. Elucidate the salient pointsof the agreement and its shortcomings.

Dear Aspirants,

The Current Affairs Questions and Answers section contains 16 questions having current relevance, with theirmodel answers to provide you a good understanding of such questions in UPSC Civil Services (Mains) exam.Answer writing in mains requires grip over both content and presentation and that too under the constraint of timeand word limit. The best way to improve answer writing is through regular practice and these model question-answers are intended to guide you in tackling such questions. We look forward to your feedback and comments.

Regards, Team Drishti

Page 47: CA June 2016 Eng Xaam.in

8/16/2019 CA June 2016 Eng Xaam.in

http://slidepdf.com/reader/full/ca-june-2016-eng-xaamin 47/68

Potential Q&As

174 || Drishti Current Afffairs Today || June 2016

Answer:  The 'one in, one out' dealbetween the European Union (EU) andTurkey seeks to address theoverwhelming illegal migration ofrefugees and asylum seekers intoEurope from Turkey.

Key points of the agreement:

  All new irregular migrants crossingfrom Turkey into the Greek islandsafter March 20, 2016 will be returnedto Turkey.

  For every Syrian refugee beingreturned to Turkey from the Greekisland, another Syrian will beresettled in EU.

  Visa liberalization for Turkishnationals will be accelerated.

  Financial support for Turkey'srefugee population will be boosted.

  The process of accession of Turkeyinto the EU will be fast-tracked.

The deal has been criticised forbeing legally, morally and practicallycontentious. Also, it is alleged that itswording have been left deliberatelyopaque in many areas. Legally, thedeal is said to contravene the 1951Geneva Convention which states that

refugees should be assessed onindividual basis, rather than beingsubject to a blanket policy as is beingdone under this deal.

The agreement morally andethically puts the EU in a tight spot asit may end up violating its own lawsand standards with respect totreatment of asylum seekers. Further,by focussing only on Syrians the dealdiscriminates against people likeAfghanis and Iraqis, eeing otherwarzones and dictatorship but whohave not received the same level ofmedia attention. By regulating themigration across the Aegean Sea, thedeal simply encourages the refugeesto enter EU through different andpotentially more dangerous routes.Also, the huge logistical challenge inimplementing the agreement fully,risks making it irrelevant to addressthe migrant crisis effectively.

Ques.4: Discuss the new loan pricingmechanism Marginal Cost of Fundsbased Lending Rate (MCLR) adopted

by R.B.I. and explain its relevance inthe economic cycle?Answer: Marginal Cost of Funds based

Lending Rate (MCLR) system is a newuniform methodology to ensure fairinterest rates to both borrowers andbanks. It will address the primaryobjective of expediting transmissionof benets of policy rate cuts by R.B.I.to end-consumers by banks. It willbring transparency and uniformity inthe methodology adopted forcalculating lending rates. This has beendone in view of banks showingreluctance in changing lending and

deposit rates in accordance with thechange in policy rates by R.B.I.

MCLR Regime

MCLR makes it mandatory forbanks to review their lending ratesperiodically to reect changes in thecost of borrowing periodically. TheMCLR is to be computed on bank’smarginal cost of borrowing instead ofthe average cost of outstanding fundsthat banks have used so far.

The components of MCLR are asfollows:

  Marginal cost of funds

  Negative carry on account of CRR(it is the cost that the banks have toincur while keeping reserves withR.B.I as CRR; the cost of such fundskept idle can be charged from loansgiven to the people)

  Operating Costs

  Tenor Premium

Benets from using MCLR

  It will reduce the pressure on bank’snet interest margins (NIMs).

  For customers lower interest ratesand decline in EMIs on their loans.

The MCLR regime will allow thebanks to become more competitive andenhance their long term value andcontribution to economic growth.

Ques.5:  In what way could powerministry’s Ujwal DISCOM AssuranceYojna (UDAY) help in turnaroundand revival of Power DistributionCompanies? Discuss.Answer: The weakest link in the powersector is distribution and state powerdistribution companies (DISCOMS)have been facing extreme nancialcrunch due to various reasons such astransmission losses, tariff hike not intandem with the rising costs andpower theft etc.

The UDAY scheme focuseson four major areas

  Improving the operat ionalefciencies of DISCOMS.

  Reduction of power generation cost.

Reduction in interest costs ofDISCOMS.

Imposing nancial discipline onDISCOMS.

Provisions of the scheme

Under previous schemes therewere no mechanism to checkunsustainable borrowing, UDAYrecognizes this loop hole; it permitsrenancing of the losses of DISCOMSonly via DISCOM bonds guaranteedby the state government.

75% of the DISCOM liabilities will betaken over by the state governmentover a period of ve years enforcingdiscipline on states as it requiresthem to absorb a part of the futurelosses of the DISCOMS in gradedmanner.

DISCOM debts not taken over bythe state will be converted by banksinto loans or bonds with minimalinterest rates.

As an incentive, the states adheringto the operational milestonesof the scheme will be givenadditional funding through variousprograms like IPDS, Power SystemDevelopment Scheme along withincreased hours of power supply.

On the revenue side it has provideda nancial road map to bring tariffsin line with costs by Financial Year2019; mandates like strict adherence

Page 48: CA June 2016 Eng Xaam.in

8/16/2019 CA June 2016 Eng Xaam.in

http://slidepdf.com/reader/full/ca-june-2016-eng-xaamin 48/68

Word ‘academic’ evokes mental imagery related to theory,seriousness, university, higher education, learning and scholarship.

Vitamins are organic compounds needed for normal growth and

nutrition. ‘Academic vitamins’ is the section of the magazine where

we give you synopses of three academically oriented magazines

which have a reputation in the UPSC ecosystem: Economic and

Political Weekly, Yojana (ISSN 0971-8400; a development monthly

published by Ministry of Information and Broadcasting) and

Kurukshetra (monthly journal of rural development by the Ministry

of Rural Development).

Economic and Political Weekly (EPW; ISSN 0012-9976), the

Sameeksha Trust Mumbai-based publication, has been ‘India’spremiere journal for comments on current affairs and research in

social sciences’. “EPW has always been receptive to those who

question the dominant power, dominant beliefs and dominant

ideas...,”says its outgoing editor C. Rammanohar Reddy. If you

want to know the different sides of a story (and especially the sides

which get ignored by the dominant narrative) then you should

read EPW regularly. EPW is very useful resource for both would-be and serving bureaucrats who want to build

balance of judgement and intellect in their arguments and writings. You are supposed to be a catalyst for social justice,

equality, and freedom, something that the UPSC will test you on. Reading EPW should adequately prepare you on

that front. We give you synopses of four issues from the last month in our monthly magazine. We lter information

from academic journals and present them lucidly to you so that you have ready-made inputs that you can use forexam preparation. We are sure you will like the EPW academic inputs and benet from reading them.

YOJANA: Since the pattern of UPSC exam has been tilting towards the awareness and analysis of public life

events, social and political happenings and trends, Yojana assumes a signicant importance because of its credibility,

authenticity and clear analysis of such events, coming from the desks of government ofces (Ministry of Information

and Broadcasting). Despite this, students nd it difcult to keep abreast with all its issues, given the paucity of time.

Hence, to simplify it for you we have made a gist of Yojana which will enable you to quickly grasp the important

points and will be useful for a quick revision just before the exams, to help you make your answers more substantive

and score more. Hope the experience proves enriching, satisfactory and fruitful for you.

KURUKSHETRA: Ministry of Information and Broadcasting, GOI brings out Kurukshetra, a monthly journal

on Rural Development highlighting challenges, status and solutions to ‘Empower Bharat’. It constitutes of articles

written by intellectuals, on-eld activists and civil servants, comprehensively describing recent policies of government,innovative steps by local administration, model cases, in the elds of agriculture, small-scale industrial development,

effective service delivery etc.

In this section, we have attempted to provide you a synopsis of important, exam-oriented topics from Kurukshetra

issue of last month. An organized structure, smooth-owing summary, lucid language without overburdening you

with facts and gures will denitely help you in the pursuit of answer writing.

In Mahabharata, Kurukshetra was the sacred land where ancient intellect once ourished. In today’s Kurukshetra

synopsis, we aim to nourish the intellect of aspiring bureaucrats. It will not just instil a rich knowledge base in you

but also develop skills of critical analysis and inter linking as demanded in civil service examination. We look forward

to a wonderful cooperation with you and wish you a very Good Luck in achieving your dream of becoming a successful

civil servant.

 ACADEMIC VITAMINSGIST OF ISSUES OF EPW, YOJANA, KURUKSHETRA

Page 49: CA June 2016 Eng Xaam.in

8/16/2019 CA June 2016 Eng Xaam.in

http://slidepdf.com/reader/full/ca-june-2016-eng-xaamin 49/68

180 || Drishti Current Afffairs Today || June 2016

ECONOMIC AND POLITICAL WEEKLYGist of 26 March, 2 April, 9 April and 16 April, 2016 issues in that order.

Summary of editorials followed by commentaries, special articles, discussions etc.

Address the StigmaAttached to HIV/AIDS

  EPW editorial laments that notenough was being done to addressthe stigma attached with AIDS andHIV.  Akshara R, an HIV-positivestudent in Kannur had to ght toremain in an educational institution.India has long list of children and

youth who were thrown out ofinstitutions and socially ostracizedfor being HIV positive. Authoritiessay that it is almost always the ‘other

 parents’ who want the HIV-positivechild to be told to leave as they do notwant their kids to mix with them.

  India’s rst AIDS case was diagnosedin 1986; India has about 2.1 millionliving with HIV. Social activistsworking in the eld remark thatthe social stigma attached to it is

more baneful than the virus.  According to National Family

Health Survey (NFHS-4, January2016), about 82% women and 70%of men (in the 13 states where thesurvey was done) do not have muchknowledge  of HIV/AIDS and safesex practices.

  Since HIV/AIDS is associated withsex; it fuels taboos and prejudices.It calls for awareness campaignstaking into account socio-culturalnorms and attitudes.

  National AIDS Control Organisation(NACO) has faced fund cuts andis no longer an autonomous body.It directed funds to states’ AIDScontrol bodies but now the stateshave discretion to fund theirHIV related programmes. On 1December 2015 (World AIDS Day),Maharashtra AIDS officials hadreported lack of funds.

  Political and religious leadersemphasizing individual moralityand abstinence while denouncingcondom usage is detrimental forAIDS control as moralising createsbarriers and prevents reach to mostvulnerable sections such as sexworkers, men having sex with men,victims of sex trafcking, women

with HIV-infected husbands whocan’t walk away from marriagedue to socio-economic conditions.

ICC Recognises Rapeas a War Crime

  Former vice president of DemocraticRepublic of Congo (DRC),  JeanPierre Bemba, was held guilty ofusing rape as a weapon of war bythe International Criminal Court

(ICC) in March. It is a rst for theICC: rst ever conviction for sexualcrimes as constituting a war crime.It was an uphill task; it took somefour years as 77 victims testiedout of an estimated 5,000 who wereidentied as survivors.

  Bemba was commander of his1,000 troops who pillaged, rapedand murdered thousands overve months in the Central AfricanRepublic (CAR) in 2002-03 where

they went to prevent a coup.Chief prosecutor of ICC said, “…commanders are responsible for theacts of forces under their control.” 

  The ICC has global jurisdiction butit is limited to hearing cases that theUNSC refers to it or from citizensof any of the 123 countries  thathave signed and ratied the RomeStatute. India, Pakistan, Indonesia,Israel, Russia, Egypt, Turkey, Iran 

are some prominent countries thathave not ratied it.

ICC has heard cases of war crimes,mostly in Africa, since it started in2002. A 2011 study says that 12%of women in Congo have been raped atleast once in their lives. ICC has beenempowered by UNSC Resolution1820 of 2008 “rape and other forms

of sexual violence can constitute warcrimes, crimes against humanityor a constitutive act with respectto genocide.”

  The conviction definitely sets alegal precedence and it is hoped itwill bring justice to women aroundthe world including in formerYugoslavia and Rwanda which sawsimilar horrors.

Vigilantism

  EPW writes that the Governmentof Chhattisgarh and the Centrehave entered a new phase in theirwar on the Maoists. State policehas initiated a new incarnation ofthe Salwa Judum, thereby violatingthe Supreme Court’s directionsbanning vigilante groups. Reactingto it, Maoists have stepped up theirretaliatory violence, including thekilling of informers and bombings.

On 30 March, they killed seven jawans of CRPF.

  Salwa Judum began life as the  Jan Jagran Abhiyan  in which policeassembled villagers and forced themto inform on sangham membersor Maoist sympathizers. Thesesangham members were made tosurrender, and some of them werearmed and appointed as SpecialPolice Ofcers (SPOs).

Page 50: CA June 2016 Eng Xaam.in

8/16/2019 CA June 2016 Eng Xaam.in

http://slidepdf.com/reader/full/ca-june-2016-eng-xaamin 50/68

Academic Vitamins

Drishti Current Afffairs Today || June 2016 || 181

  T h e r e h a s a l s o b e e n t h eannouncement of the formationof gram rakshak dals or villagedefence committees which haveproved disastrous for civil solidaritywherever they have been tried, inNagaland or Kashmir.

Government must realize thatvigilantism is a Frankensteinmonster that kills its own sponsors.

Nuclear Reactor Leak

On 11th March, the fth anniversaryof the multiple reactor accidents atthe Fukushima Daiichi site in Japan,Unit 1 of the Kakrapar Atomic PowerStation (KAPS-1) in Gujarat startedleaking heavy water, which could

be plugged only by 21st March afterstrenuous efforts.

  Even though the leak at KAPS-1 isnot a new thing (such leaks havebeen regular occurrences eversince the rst heavy water reactorin Rajasthan came online), publicinformation about them is scarce.There is enough evidence from thereports put out by the InternationalAtomic Energy Agency (IAEA) thatsuch leaks have been occurring at

most Indian heavy water reactors,including at the Kakrapar I &II, Rajasthan I & II, Madras I &II, and Narora I & II reactors.The persistence of leaks suggestfundamental and unresolvable awsin the system, technical weaknessesin the regulation, or continuedoperator errors, pointing towardslack of a good safety culture, andraising concerns about the possibilityof a severe accident in the event of

a greater challenge (for example,in the event of a re or ooding).

  Such leaks lead to increasedradiation exposure to workers atthe nuclear plants: the heavy waterbecomes radioactive as deuteriumabsorbs neutrons produced duringssion and becomes tritium whichis easily absorbed by human body.

  In the long term, secrecy on theworking of atomic power plants

will not be conducive to safety. Itis important to ensure operationalsafety of power plants; government’smethod of paying lip service tosafety measures will not be of help.

Misuse of Tax Havens  EPW writes that the Panama papers

have exposed the ugly underbellyof globalization. However, artfullawyers and accountants defendtax havens as being neither evilnor illegal.

The editorial writes that not muchis clearly known about 90-odd taxhavens in the world today (includingthose within the domestic boundary

of a country such as Delaware andFlorida in the US or principalitiesor microstates like Monaco andLiechtenstein in Europe).

Money moving rapidly acrossmultiple tax jurisdictions (whichis called ‘round-tripping’ ) makesthings hard for authorities as thedetermination of ‘benecial owners’of companies gets complicated.

  There is a thin line between legaltax avoidance and illegal forms of

tax evasion.

Medical Overuse Crisis

  There may be a ‘medical overuse’crisis in India as pointed out byhealth activists, medical journalsand the World Bank.

A recent media report indicatesthe number of surgeries done inthe ve years between 2009-10 and2014-15; number of major surgeries

conducted under the NationalHealth Mission (NHM) has goneup considerably. Caesarean section,hysterectomies and other emergencysurgeries increased by 979% inMaharashtra, 470% in Karnataka,400% in Bihar. ‘Poor have surgeriesbut not healthcare,’  opines EPW.

  Health activists in Andhra Pradeshhad showed that avoidableC-sections were being done to

claim money under  Arogya sr i government scheme. Chhattisgarh’s‘uterus scam’ (women subjected tohysterectomies) had got internationalmedia attention.

The WHO has clearly said that

C-section should be a life-savingmeasure and incidences should be15%-20% for a region.

  Some useful measures to arrest thetrend of quick-re surgeries could bea more proactive Indian MedicalAssociation (IMA), spreading publicawareness, checking malpracticesin private hospitals and strictmonitoring of rules.

Maharashtra: Water

Management, Not WaterScarcity is the Problem

  The Bombay HC order on 13 Aprilin response to a PIL, orderedIPL matches out of drought-hitMaharashtra from May. It does littleto address the root of the problem.The problem faced by Maharashtraand nine other states is also a crisisof good governance.

  Two consecutive monsoon failures

don’t explain the water crisis; it isthe failure of governments in providingsustainable water resource management.Things will be ‘business as usual’once ‘good monsoon’ comes thisyear; whereas in scarcity we worryabout preventing wastage andseeking emergency supply.

  Maharashtra has 1,845 dams whichis more than any other state inIndia. Still, only about 18% ofits farmland is irrigated. A ‘damscam’  exposed how dams were builtwithout giving irrigation benets.There are 70,000 minor irrigationprojects in the state, but only 12%are working today.

‘Sugar lobby’ in Maharashtra hasused its clout to dictate surfacewater for irrigation; water-intensivesugarcane cultivation was neverchecked by any government. 80 ofthe state’s 205 sugar factories are

Page 51: CA June 2016 Eng Xaam.in

8/16/2019 CA June 2016 Eng Xaam.in

http://slidepdf.com/reader/full/ca-june-2016-eng-xaamin 51/68

Academic Vitamins

188 || Drishti Current Afffairs Today || June 2016

The North East India

  It comprises of eight states: - Assam,Tripura, Meghalaya, Mizoram,Nagaland, Arunachal Pradesh,Manipur and Sikkim, forming thenortheast frontier of our country.

  The entire North East Region(NER), approximately 90%, is aninternational border shared with

China and Bhutan in the North,Myanmar in the east, Bangladeshin the south and west, and Nepalto the west of Sikkim.

  It is home to various ethnically,culturally, linguistically andreligiously diverse populationinhabiting geographically diverselandscapes. It is also one of thehotspots of India because of itshuge biodiversity and presence ofendemic species.

  The region faces socio-politicalchallenges due to insurgencies,terror i sm, tr iba l conf l i c ts ,insider versus outsider tensions,smuggling, poor infrastructure,floods, lack of educational andemployment opportunities leadingto outmigration.

Inclusive Growth andRegional Economy

  The North East region can be openedup to trade with entire South EastAsia by opening up its regionaleconomy through improved tradeand political relations with itsneighbor Bangladesh.

  The ‘ease of doing business’ in theregion has been improving, as canbe seen in Tripura (Agartala), wherethe State Government has got amajor Internet Gateway (India’s

third gateway after Mumbai andChennai),which will improvetelecommunications of North Eastby connecting it to reliable globaltelecom network.

  One of the reasons for Tripura

becoming the trendsetter in

providing the ‘ease of doing

business’ climate is that, it has

overcome insurgency and AFSPAhas been recently uplifted in

2015, providing a stable business

environment.

  Even the Bangladesh government

has shown its support by openingup consular ofces in Bagdogra,

Sylhet and Guwahati to improve

connectivity and reduce official

hassles. Presently, only Agartalaand Kolkata have consular ofces.

Challenges/limitations to

growth and economy

  The image of disturbed area, due to

60 years of conict and insurgency,

has been big hurdle in invitingprospective business opportunities.

  Land-locked Region:  the regionhas lost out the policy of ‘connect

to SE Asia’ to the coastline states ofWest Bengal, Andhra Pradesh and

Tamil Nadu, because internationaltrade is predominantly moved by

sea. The problem can be solved by

improving the region’s connectivity

and enabling its access to Bay ofBengal, connecting it through rail

line from Agartala via Akhaura in

Bangladesh to Chittagong Port.

  Poor infrastructure  has also

been a factor because of difcult

topography.

 YOJANA Summary of the Yojana issue of month April (2016) – North-East Special

  Regional growth defcit of northeast vis-à-vis better performingstates of Tamil Nadu, Maharashtra.

  The region has essentially beena “market” and not a productioncentre, importing most of thenished products. Raw materialslike fruits, vegetables and even cattleare being exported to Bangladesh

and Myanmar as there have been nomajor processing units for addingvalue or shelf life to the products,hence impacting the economy.

  Poor human development indicators (Assam recording worst MaternalMortality Rate of 300, far aboveeven U.P., Bihar and M.P.)

Changing Climate Patterns leadingto frequent oods and water logging.In absence of any adequate inlandnavigation system, transport suffersfor 3 to 5 months every year affectingeconomy.

Education and Employment

  The Northeast region has a literacyrate of 77.76% which is higher thanthe national average of 74.04%.Despite this, education system innorth east suffers from variouslimitations.

Challenges/limitationsin education andemployment sector

  The education system is controlledby the state government which is notat par with the national level. TheStates are still following traditionalmode of imparting education. Thishas resulted in huge outow andhigh drop-out rates of studentsbetween Class 1 to 8.

Page 52: CA June 2016 Eng Xaam.in

8/16/2019 CA June 2016 Eng Xaam.in

http://slidepdf.com/reader/full/ca-june-2016-eng-xaamin 52/68

Academic Vitamins

Drishti Current Afffairs Today || June 2016 || 189

  The bookish curriculum and lackof technical/ practical exposure hasled to low employability.

   It also suffers from exogenous factorslike numerous bandhs and strikes(compelling parents to send their

children outside, sometimes at hugepersonal costs like child trafcking),insurgency, cross border smuggling,terrorism, sociopolitical unrest,oods, lack of basic necessities likeelectricity, water, internet, transport,communication etc.

The comparative advantageof North East states lies in

l  Large English speakingpopulation.

  l  Culturally diverse mix ofstudents.

  l  High literacy rate.

The way forward

  The focus should be on industrytraining, quality over quantity,r es ea r c h a nd enc o u r a g i ngentrepreneurship.

  Corporate Companies can be askedto engage in equipping technical

and non technical institutions underCSR (Corporate Social Responsility)initiative to increase employabilityrate.

  C r e a t i o n o f e m p l o y m e n topportunities through initiativeslike  Make in India  and Start-Up models.

  The government departments atdifferent levels like ICC, FICCI,FINER and the civil society have

to come together to transformnorth east into an educational andtourism hub.

  There is a need of multiprongedapproach like:-

  l  Fast tracking of infrastructureprojects.

  l  Connecting major tourist spots.

  l  Incentivizing trekking andecotourism to make northeasthub of adventure tourism.

  l  Strategy to promote community

based collaterals for effectivecredit delivery system.

l  Establishment of SEZs (SpecialEconomic Zones) to captureeconomic advantages in tea,

coffee, aromatic and medicinalplants, horticulture products.

  l  Strong R&D support system.

  l  Efcient market system witheffective supply chain modelwith help of specializedinstitutions like IIM Shillong.

Above all, the presence of politicalwill, bureaucratic support andcommitment, good governance, easeof doing business and PPPP (PrivatePublic People Participation) Model

will prove signicant in improvingeducation and employability in NorthEast India.

Skill Development

  The Centre is planning to takeup its Skill India initiative to theNorth East (NE) by setting up SkillDevelopment Centres and IndustrialTraining Institutes (ITIs)  in newdistricts.

  The Union Skill Development andEntrepreneurship  minister hasrecommended inclusion of onemember each, for all the 40 SectorSkill Councils, from Federation ofIndustry and Commerce of NorthEastern Region (FINER) and hasproposed to restructure NationalSkill Development Corporation(NSDC) and include FINER as amember.

  The NE is growing fast, with high

literacy rate, reducing dependenceon agriculture, but there is a lagin creation of jobs and livelihoodopportunities, creating a hugemismatch. According to a studyon development and employmentgeneration potential of NE states,between 2011 and 2021, the regionwill have only 2.6 million jobs asopposed to supply of 17 millionpeople in the job market.

  To address this mismatch andfor infrastructure development,Centre had created the Ministryof Development of North EasternRegion (DoNER), in 2004.

  While addressing the employment

requirements, the challenges areencountered in poor infrastructureand connectivity, law and orderproblems, quality of skills (only4.5 persons out of 100 are skilled)of the available labour pool.

Paryatak Mitra Programme

The programme has beenlaunched by Ministry of Tourism forcollege going students to inculcateappropriate tourism traits andknowledge and enable them to act/work as Tourist Facilitators(Paryatak Mitra). Even though thedevelopment and promotion oftourism is primarily under StateGovernments, the Ministry ofTourism, as part of its ongoingpromotional activities releasescampaigns and undertakes activitiesunder the  Incredible India brand-line to promote India as a holistictourist destination.

The Way Forward  A Skill University  in the North-

East needs to be urgently set up toaddress the aspirations of youth.

  Migration Support Centresshould be set up for better support,retention and career opportunitiesfor candidates. This measure will bemore essential for trainees comingfrom North-East, hilly states andother difcult areas like LWE (LeftWing Extremism) affected districts.

  Employment opportuni t iescan be created in Meghalaya inagarwood plantations, learningthe business model from SouthEast Asian countries where avertically integrated business iscreated, that includes inoculation,harvesting, distilling and processingof agarwood into various endproducts (including the highly-prized Oud oil).

Page 53: CA June 2016 Eng Xaam.in

8/16/2019 CA June 2016 Eng Xaam.in

http://slidepdf.com/reader/full/ca-june-2016-eng-xaamin 53/68

Academic Vitamins

Drishti Current Afffairs Today || June 2016 || 197

KURUKSHETRA 

In this section we bring you a summary of important topics from Kurukshetra (Journal on RuralDevelopment) April 2016 Edition, which provides important features of Budget 2016-17.

Rail Budget 2016-17:

Connecting India,

Creating Jobs

Indian Rail is not just a mode of

communication but Life Line of our

Country. It has multiple dimensions-

Cultural, Social, Economic, religious.

Fund Allocation  1. Budget-2016 allocates 1.21 lakh

crore for Capital Expenditure

which is close to the double

of average of previous years

– a feat never achieved before.

  2. Railway suffers from high

revenue expenditure due to

High Operating Ratio (92%)

thanks to a bloated bureaucracy

of about 13 lakh employees

and recent 7th Pay Commission

hike. Still there is hope to

generate revenue 10.1% higher

than revised target for current

year because of optimistic

global predictions, falling oil

prices and expansion in

business activities.

Vision of Indian Railways

By 2020, Budget-2016 looksforward to meet the long-felt desiresof the common man:

  1. Reserved accommodation ontrains being available ondemand.

  2. Time tabled freight trains withcredible service commitments.

  3. High end techn ology tosignicantly improve safetyrecord.

  4. Elimination of all unmannedlevel crossings.

  5. Punctuality increased to almost95 percent.

  6. Increased average speed offreight trains to 50 kmph andMail/Express trains to 80kmph.

  7. Semi high speed trains runningalong the golden quadrilateral.

  8. Zero direct discharge of humanwaste.

How to achieve theVision of Budget 2016

Moving by the ideology of ‘SabkaSaath, SabkaVikas’, the budget-2016

focuses on common man’s comfort.Plans envisaged are as under:

 In Social Sector 

  Ensure social justice: Reservationhas been allowed in Catering unitsfor SCs, STs, OBCs, etc. PersonalProtective Equipment (PPE) forsanitizing workers will improvetheir working conditions, use of Bio-toilets will lessen the requirementof manual scavengers itself (in linewith Manual Scavenging AbolitionAct).

   Women Empowerment:  A subquota of 33% for women has beenenforced in each of the reservedcategories of catering.

  Skill Development: Indian Railwayshas tied up with Ministry of SkillDevelopment to train rural youth.

It will also collaborate with Khadiand Village Industries Commissionto develop consumer products likebed sheets, cushion covers, etc. forpassengers and also market themusing IR’s vast Supply Chain.

  Local Employment:  Weightagewill be given to district domicileholders for commercial licenses

Major Reforms envisaged in Budget-2016 are provided in the mindmap.

Page 54: CA June 2016 Eng Xaam.in

8/16/2019 CA June 2016 Eng Xaam.in

http://slidepdf.com/reader/full/ca-june-2016-eng-xaamin 54/68

Academic Vitamins

198 || Drishti Current Afffairs Today || June 2016

to operate stalls, shops, services atlocal stations. (in line with Standup/Start up India Mission).

  Health Benefits:  Provision forBio-Toilets, Swachh Bharat Abhiyan,standard operating guidelines for

catering services such as propercleanliness in kitchen, hand gloveswhile cooking etc. (They are in linewith provisions of Draft NationalHealth Policy-2015 which aims tohave quality cleanliness standardsat public places.)

  Environment Conservation/ Mitigation and Adaptation:  IRhas taken up measures to combatGlobal Warming, Climate Changeby adhering to Sustainable

Development Standards likeRailway electrication, water recycle& reuse, outsourcing afforestationprojects, installing solar panelson rooftops and on huge tracts offallow land available adjacent torail networks etc. (This will be inline with National Action Plan forClimate Change–NAPCC-2008 andParis Agreement as well).

 In Economic Sector 

  Budget-2016 aims at better cateringfacilities with local cuisines availableon board. Provision has been madeto collaborate with start-ups, SHGs,e-retailers, local Food ProcessingUnits to supply local cuisines toonboard customers. This will notonly establish a unique identity ofthat local village, export its cultureto the passing passengers butalso generate in situ employmentopportunities  for the increasingvillage youth. This will create awin-win situation of passengersatisfaction and rural development.

  National Rail Plan (NRP-2030) will integrate all stakeholderslike state governments, publicrepresentatives, central ministriesto synchronize Rails with othermodes of transport, creating aseamless multimodal transportationnetwork  across the country. This

plan will reduce freight delays,thereby increasing ‘Ease of DoingBusiness’ in India, pushing growthinto double digits as envisaged byNational Manufacturing Policy-2012.

  Indian Railways aims at operating

Tourist Circuits with cooperationfrom state governments whileimplementin g schemes likeHRIDAY, PRASAD. This will havea multiplier effect on other sectorsas well. For e.g. hotels, restaurants,hospitals, travel agencies, adventuresports etc. will come up.

Indian Railway Budget 2016envisages a transformation from scale

to speed. It aims at Customer

Satisfaction and at cultivating a feelingof ownership among citizens followedby satisfaction eventually to pride andnally culminating in Nationalism.

A Boost to Organic Farming 

Organic Farming means use ofonly naturally occurring products andpractices in agriculture such as cowurine, wood ash, compost andbio-resources.

Status of Organic

Farming in India

  Cultivated area under certified

organic farming has grown almost17 fold in past decade.

  Last year, India produced around1.24 million Metric Tonnes  ofcertied organic products such assugarcane, cotton, maize, wheat,rice, peas, etc

Distribution Patterns

  North Eastern states lead inpercentage of land under organiccultivation.

In absolute terms, Madhya Pradeshleads followed by Himachal Pradesh

and then Rajasthan.

Problems faced by organicfarming 

Despite immense potential,cultural best-practices in India, thereare many structural challenges asgiven in table.

Steps taken by GOI

 Paramparagat KrishiVikas Yojna (PKVY)

It aims at building Clusters oforganic farms with size of 50 acres ormore comprising 50 or more farmers.Farmers will be incentivised by cheapcredit, subsidised bio-products,guidance and other facilitation. Targetin next 3 years is to have 10,000 clustersformed, covering 5 lakh acre areaunder Organic Farming.

 National Programme for

Organic Production (NPOP)It involves an accreditation

programme for Certication bodies,specifying quality standards and toenhance organic worth of our productsto match international standards.

Organic Value Chaindevelopment in North East Region

The scheme aims at development

of certifed Organic production in a

Input Farming sidechallenges

Transportation challenges Marketing sidechallenges

Lack of private investment. Poor cold storage facility. Branding and

C e r t i f i c a t i o nchallenges.

Small land holdings. Poor food processing sectorsuch as packaging, grading,sorting, logistics, etc relatedactivities.

Middle mendelays, APMCAct, etc.

Less cooperatization.

Page 55: CA June 2016 Eng Xaam.in

8/16/2019 CA June 2016 Eng Xaam.in

http://slidepdf.com/reader/full/ca-june-2016-eng-xaamin 55/68

Ethics, Integr ity & Aptitutde

202 || Drishti Current Afffairs Today || June 2016

“When our emotional health is in abad state, so is our level of self-esteem. Wehave to slow down and deal with what istroubling us, so that we can enjoy thesimple joy of being happy and at peacewith ourselves.”  - Jess Scott

Emotional intelligence (EI) is theability to monitor one’s own and other

people’s emotions, to discriminatebetween different emotions and labelthem appropriately, and to useemotional information to guidethinking and behavior.

In simple words, EmotionalIntelligence refers to attributes suchas understanding one’s feelings,empathy for others, and the regulationof emotions to enhance one’s life.

For instance, in an organizationwhen an employee is faced with a

challenging situation, she can handleit in either of the following two ways:

  (a) She might see the challengingsituation as an opportunity toprove herself or;

  (b) She might become nervousand experience fear of failure.

Background toEmotional Intelligence

Aristotle wrote about emotionalintelligence in 350 BC, centuries beforethe term became popular. In 1983,Howard Gardner’s “Frames of Mind:The Theory of Multiple Intelligences”introduced the idea that traditionaltypes of intelligence, such as IQ, failto fully explain cognitive ability. Heintroduced the idea of multiple

intelligences which included bothinterpersonal intelligence (the capacityto understand the intentions, motiva-tions and desires of other people) andintrapersonal intelligence (the capacityto understand oneself, to appreciateone’s feelings, fears and motivations).

Working on your Emotional Intelligencecould well be the most important aspectof your personal development. Researchhas shown that people with higher levelsof emotional intelligence enjoy moresatisfying and successful careers andrelationships. If you think about waysto enhance your Emotional Intelligence,you are likely to become more interestingand attractive to others, and you willalso give your self-esteem a boost.

The rst use of the term“Emotional Intelligence” is usuallyattributed to Wayne Payne’s doctoralthesis, “A Study of Emotion:Developing Emotional Intelligence”(1985). After this in 1990s, twocolleagues from Yale University, PeterSalovey and John Mayer wrote acomprehensive research essay onEmotional Intelligence. However, theterm became popular and widelyknown with the publication of Daniel

Goleman’s groundbreaking bestseller“Emotional Intelligence – Why it canmatter more than IQ” (1995).

Five Components ofEmotional Intelligence

According to Daniel Goleman, anAmerican psychologist who helped topopularize EI, there are vecomponents critical to emotionalintelligence:

Emotional Intelligence:Moving beyond IQ

Know YourEmotions

ManageYour

Emotions

Recognize& knowOthers'

Emotions

Managethe Emotions

of Others

MotivateYourself

EmotionalIntelligence

ETHICSGS Paper IV

Page 56: CA June 2016 Eng Xaam.in

8/16/2019 CA June 2016 Eng Xaam.in

http://slidepdf.com/reader/full/ca-june-2016-eng-xaamin 56/68

Ethics, Integr ity & Aptitutde

Drishti Current Afffairs Today || June 2016 || 203

  1. Self-awareness: The ability torecognize and understandpersonal moods and emotionsand drives, as well as theireffect on others. Hallmarks ofself-awareness include self-

confidence, realistic self-assessment, and a self-deprecating sense of humor.Self-awareness depends onone’s ability to monitor one’sown emotional state and tocorrectly identify and nameone’s emotions.

  2. Self-regulation:  The abilityto control or redirect disruptiveimpulses and moods, and thepropens i ty to suspend

 judgment and to think beforeacting. Characteristics includetrustworthiness and integrity;comfort with ambiguity; andopenness to change.

  3. Internal motivation: A passionto work for internal reasonsthat go beyond money andstatus -which are externalrewards, – such as an innervision of what is important inlife, a joy in doing something,curiosity in learning, a ow

that comes with beingimmersed in an activity. Apropensity to pursue goalswith energy and persistence.Attributes include a strongdrive to achieve, optimismeven in the face of failure, andorganizational commitment.

  4. Empathy:  The ability tounderstand the emotionalmakeup of other people. A

skill in treating peopleaccording to their emotionalreactions. Characteristicsinclude expertise in buildingand retaining talent, cross-cultural sensitivity, and service

to clients and customers.It is important to note that empathydoes not necessarily implycompassion. Empathy can be ‘used’for compassionate or cruel behavior.Serial killers, who marry and killmany partners, tend to have greatempathic skills!

  5. Social skills: Profciency inmanaging relationships andbuilding networks, and ability

to fnd common ground andbuild rapport. Hallmarks ofs o c i a l s k i l l s i n c l u d eeffectiveness in leading change,persuasiveness, and expertisebuilding and leading teams.

Emotional Intelligence,Intelligence Quotient,and Personality

Emotional intelligence taps intoa fundamental element of human

behavior that is distinct from yourintellect. There is no known connectionbetween IQ and emotional intelligence;you simply can’t predict emotional

intelligence based on how smartsomeone is. Intelligence is your abilityto learn, and it is the same at age 15as it is at age 50. Emotional intelligence,on the other hand, is a exible set ofskills that can be acquired andimproved with practice. Althoughsome people are naturally moreemotionally intelligent than others,you can develop high emotional intelli-gence even if you aren’t born with it.

PERSONALITY

Personality is the nal piece of thepuzzle. It is the stable “style” thatdenes each of us. Personality is theresult of hard-wired preferences, suchas the inclination toward introversionor extroversion. However, like IQ,personality can’t be used to predictemotional intelligence. Also like IQ,personality is stable over a lifetime and

doesn’t change. IQ, emotionalintelligence, and personality eachcover unique ground and help toexplain what makes a person tick.

Page 57: CA June 2016 Eng Xaam.in

8/16/2019 CA June 2016 Eng Xaam.in

http://slidepdf.com/reader/full/ca-june-2016-eng-xaamin 57/68

210 || Drishti Current Afffairs Today || June 2016

–Tapesh Kumar 

The paths to glory lead but to the grave

This section of the magazine gives you essays that you can read to gain insight on essay writing skills and alsoknowledge. Also check out and participate in the Essay Writing contest and win cash prizes as you improve your writingskills.

There is an astonishing dispositionin human mind to seek pleasure fromvaried sources, even from pain andmelancholy. If it weren't so, the gothicgenres of literature, horror and bloodymovies, lamenting lyrics and grievousballads would not have been sopopular. We receive comfort in gloomytale, acceptance in sad sonnets andcalm pleasure from the elegies whichraise our melancholy, for no matterhow pompous our life might be, thereis always some room for bitter truthof pain and sorrow. Eighteenth centuryEnglish poet, classical scholar andprofessor at Cambridge University -Thomas Gray - wrote this mournfulpoem titled ‘Elegy written in a countrygraveyard’ in a country church burialground:

“The boast of heraldry, the pomp of power,

 And all that beauty,

 all that wealth e’er gave.

 Awaits alike the inevitable hour,

 The paths of glory lead but to the grave.” 

Such quatrains are a continuousreminder of mortality of human lifeand inevitability of death irrespective

of social position, beauty, wealth orany glory. It was the same burial sitewhere Gray was later buried.

Of all the truth, death is the most

universal, whose realisation is assured

beyond any doubt and without any

exception. One thing that man has

consistently failed at is in achieving

physical immortality. No alchemy,sacrifce, yoga and no enlightenment

or Nirvana could materialise into an

everlasting enduring life. One who has

been born, shall die, so shall one who

will be born. Entire life, people torment

themselves for money, material, love,family, health, glory etc. But all the

varieties of trouble a life might

encounter, with all its preoccupations,

fnd their submergence in death. Death

is an inevitable phenomena associatedwith life, since the moment worldly

existence of life form is realised.

Greek hero of Trojan War, the

central character and greatest warrior

of Homer's Iliad - Achilles- was tried

to be made immortal by his mother

Thetis, herself a sea-nymph. He was

dipped into the holy water by her

mother to attain immortality. However

he was killed at the end of the Trojan

War within the city of Troy by an

arrow which struck him in the heel. Itwas discovered that he was left

vulnerable at the part of the body by

which she held him while dipping into

the water, which remained dry. After

his death, all his bravery, all the

swiftness of his sword and all his

insurmountable strength reduced toa phrase of grammar called ‘Achilles

heel’ which means a weakness or

vulnerable point. Indeed death is the

ultimate truth. There is nothing that

can stop one from realization of this

truth.

Nevertheless, fear associated with

death might be optional. The fact is,

death seems scary, not because death

is scary but because life, many a times,

gets scary. The fear of death followsfrom that of life. One who is preparedto die anytime, is the one who lives

the life fully. It is in such achievementof fearlessness for life that glory

sprouts and nurtures itself. The

durability of life is often associated

with the intensity of glory achieved.

Maria Corazon Aquino was a self

proclaimed plain housewife till she

decided to run for president in

Page 58: CA June 2016 Eng Xaam.in

8/16/2019 CA June 2016 Eng Xaam.in

http://slidepdf.com/reader/full/ca-june-2016-eng-xaamin 58/68

Drishti Current Afffairs Today || June 2016 || 215

Need for this coloumn

You must be wondering about therelevance of interview coloumn soearly. Your frst reaction would be wecan have this discussion after themains exam result. What is the needto prepare for the interview now, whenall we can think of is how to qualifythe preliminary test? Such an approachis common among aspirants. Most of

the aspirants suffer from the problemof ‘adhocism’ which means aversionfrom planning, often shying awayfrom making an advance plan to tacklea situation. Candidates think of quickx solutions, though they aresuccessful in helping them tackle smallchallenges, but can’t make one sailthrough a test requiring a long,extensive and planned preparation.Personality test falls in the categoryof extensive planning as well as

exhaustive preparation.We know that a personality test

intents to weigh candidate’sperspective which cannot be examinedthrough Mains examination. Naturally,an interaction conducted to have anoverview of one’s personality cannotbe dealt with rote learning ofmagazines and notes. There is apossibility that he/she might not beable to answer a slightly twistedquestion.

In short one’s personality cannotbe overhauled overnight. It is a longand exhaustive process of learning andinculcating those changes along withpracticing them on day-to-day basis.I assume that the discussion above isenough to start preparing for theinterview. Let us begin by giving youan idea about what the interviewboard expects from the candidate.

Personality test:An intricate problem

The word 'Interview' in itself is adaunting word, which can make eventhe well-prepared candidate anxious.It is normal to feel nervous and befrightened about the interview sincethe nature of the test is different fromthat of prelims (where you choose ananswer from various options) or theMains examination. During aninterview, it is almost mandatory toanswer every question and thequestions are followed bysupplementary ones. Every wrong orvague answer may lead to negativemarking. We are well aware thatinterview does not have a prescribedsyllabus unlike the prelims and mainsexamination. Candidates fear facingthe board members whose workexperience is almost double the

aspirant’s age, so it is quite impossibleto manipulate an answer or statewrong facts in front of them. Eachmember of the panel is an expert inhis/her eld yet it is not possible forthem to judge candidate‘s perspectivein an interview barely lasting aroundtwenty to thirty minutes.

These factors make the personalitytest more of an assumptive exercise

where the panel assumes few factsabout the candidate’s personality.There are other factors in the interviewthat also matters—the assignedinterview board; panel‘s area ofinterest; their subject of expertise;candidate and the panel‘s politicalorientation; temperament of the panel;whether the panel adheres to lenientmarking or is strict and so on. All thesereasons affect your interview. Sincethere are many interview boards, thevery announcement of the panelassigned to the candidate can makehim/her feel condent or nervous.

To reduce the role of chance andthe subjective nature of an interview,a candidate must be interviewed bythree different boards. Following thisprotocol would enhance the objectivityof an interview. For language relatedissues, the government whileappointing the interview panel shouldkeep into consideration the lingual

Dear Aspirants,

We are beginning a special column to give you an insight on how to prepare for the personality test. The columnwill have three subsections comprising of mock interview as well as its evaluation. This particular section gives youthe strategy for the Personality Test. So many things could go wrong when you face the UPSC board. If you arereasonably prepared for it, you will face it condently and even direct the interview to some extent. Do rememberan interview is like a game of ping-pong; both parties often take cues from the last move directed at them.

–Vikas Divyakirti

Preparing for the Personality Test Part-I

Page 59: CA June 2016 Eng Xaam.in

8/16/2019 CA June 2016 Eng Xaam.in

http://slidepdf.com/reader/full/ca-june-2016-eng-xaamin 59/68

Name: Nivedita Narayan

Father’s Name: Rajendra Singh

Mother’s Name : Seema Singh

Place of Birth : Lucknow, Uttar Pradesh

Date of Birth : 17th July 1990

Academic Qualication:

l High School: Lucknow Public school (85%)

l

Senior Secondary: Lucknow Public School (79%)l Graduate: (B.A. Program)

Optional Subject: Philosophy

Medium:  English

Attempts: Second (didn’t qualify prelims in the rst attempt)

Interview: First

Hobbies: Painting, Reading (ction)

Service Preferences: IAS, IPS, IFS,IRS, IC&ES, IRTS, IRAS

State Preferences:  Uttar Pradesh, Madhya Pradesh, Bihar, Jharkhand,Uttarakhand, Rajasthan, Gujarat, Haryana, Punjab, Maharashtra, HimachalPradesh, West Bengal, Odisha, Karnataka, Kerela, Tamil Nadu, Andhra

Pradesh, Telangana, Assam- Meghalaya, Manipur- Tripura, Nagaland

INTRODUCTION O F THE CANDIDATE

INTERVIEW(The peon opens the door for

 Nivedita)

Nivedita: May I come in Sir?

Chairperson: Yes you may. (The

chairperson is looking at the bio data of

Nivedita while she was standing right

next to the door after entering the room.)

(Nivedita is wondering whether she

should sit or wait for the chairperson torespond... No one is looking at her and shestarts feeling drowsy because of nervousness)

Chairperson:   You are still

standing…? Please have a seat Nivedita.

Nivedita : (She greets all the members

before sitting) Thank you sir

Chairperson:   How are youNivedita?

Nivedita: I am ne sir. Thank you.

Chairperson:  So Nivedita youbelong to Lucknow, it is located on

the banks of river Gomti. Can you

briefly tell us about the course itfollows?

Nivedita: Yes sir, Lucknow is onthe banks of river Gomti. The river

originates from Pilibhit, travels

through the entire breadth of Uttar

Pradesh, and meets river Ganga atVaranasi. Till Lakhimpur Kheri it is

narrow but various tributaries join

the river here on.

Nivedita Narayan hails from Lucknow. She completed

her graduation from Lucknow University. She has been

preparing for the exam from her home town. Nivedita was

fortunate enough to enjoy the luxury of studying in a public

school and continue studies as per her wish. Though she

failed in the rst attempt of civil services examination,

Nivedita was determined to take another attempt with a

renewed vigour.

MOCK

  INTERVIEW

The section on Mock Interview is mock only in name. This interview does not happen ofcially butyou should think that it happened and take it seriously. There is no template for an interview; younever know what the board may ask you. It may become one-sided or it may be a very generalinterview. So the more versatile and prepared you are, the better it is for your future. The mockinterview will keep you battle-ready if you take it seriously. Follow the evaluation of the interviewclosely; the analysis is intended to show gaps and point out errors made in pressure inside theDholpur House and also to underscore the efcacy of various inter-personal strategies employedby the candidate during the interview to make it a natural, thought-directed, purposive conversationas far as possible.

BACKGROUND OF THE CANDIDATE

Page 60: CA June 2016 Eng Xaam.in

8/16/2019 CA June 2016 Eng Xaam.in

http://slidepdf.com/reader/full/ca-june-2016-eng-xaamin 60/68

Interview Par t Mock Interview

218 || Drishti Current Afffairs Today || June 2016

M1: Name some of the tributariesof the river?

Nivedita: I am sorry Sir, I can’trecall any name.

(Expression of fear appears on the

 face of Nivedita)

M1: Not even the name of a majortributary?

Nivedita: Sorry Sir, but I am not

able to recall the name.

(She is nervous now about not being

able to answer two questions in a row)

M1: How many cities are located

on the banks of R. Gomti?

Nivedita:  Sir…four…namely

Lucknow, Lakhimpur Kheri, Sultanpur

and Jaunpur.M1:  Government of India has

launched Namami Gange Project. Is

river Gomti included in the same?

What according to you are thechallenges that the project faces?

Nivedita:  (Thinks for some time)

Yes Sir, R. Gomti is a part of the project.One of the biggest challenges faced

by the project is that of bridging the

gap between treatment of waste water

and the capacity of treatment plants.Of the total sewage produced by the

ve states combined approximately

only half of it can be treated with the

present capacity of Common EfuentTreatment Plants (CETPs). The problem

of open defecation is specic to stateslike Uttar Pradesh and Bihar.

M1:  That’s it, according to you

these are the only problems?

(After thinking for few seconds

Nivedita answers)Nivedita:  Sir, a problem common

as well as unique among all the states

is that of cremation. Due to religioussentiments convincing people to do

away with this practice will be a difcult

task.

(M1 is waiting for her to nish)

M1: (immediately asks) So if theydon’t cremate the bodies on the river

bed, where should they?

Nivedita: Sir, alternatives like

electric or gas crematorium should

come up in adequate numbers especially

in religious cities like Varanasi andAllahabad. The government has already

ordered construction of new

crematoriums and renovation of theones that already exist.

(M1 doesn’t look satised with theanswers but indicates the next member tocontinue)

M2: I can see that your optionalsubject is Philosophy. Can I ask fewquestions in context of the subject?

Nivedita: Yes Mam.

M2: Why are Carvakas criticisedfor their metaphysics even though all

they seek is happiness?Nivedita:  (feels condent)  Mam,

Carvaka school does not believe in thetheory of God or Soul. They don’tagree with the concept of afterlife orrebirth. They simply believe in eating,drinking and making merry.......... Wecan say the ethics of the Carvaka iscrude hedonism.  It propagates theidea of seeking pleasure of the sensesonly on an individual level. Out ofthe four human values only ‘kama’ orsensual pleasure is regarded as theend, and ‘artha’ is regarded as themeans to realize the end. It outrightlyrejects the idea of Dharma and Moksha.In the realm of these reasons Carvaka’sphilosphy is also known as Materialism.

M2: Even Buddhism and Jainismhave rejected the authority of Vedas….some schools of Buddhism deny thesoul along with Materialism, so whyis Carvaka criticised extensively?

Nivedita: Carvaka school insteadof focusing on the psychological andmoral aspects of a human being wasmore concerned about satisfying thesenses. It’s denial of all human valueswhich make life worth living was notacceptable by all. Life without valuesis similar to animal life. The pursuedpleasure should be in conformancewith Dharma.

(M2 keeps a straight face)

M2: Can you throw some light on

the differences between Ramanuja’s

p h i l o s o p h y , a n d t h a t o f

Madhavacharyas’?

Nivedita: Yes Mam, Shankara

propounded advait Vedanta while

Ramanuja propogated Vishisht dvaitvada.

(M2 frowns)

M2: I think you did not hear the

question correctly. I asked for pointing

out the difference between Ramanuja

and Madhava.

Nivedita:  I am sorry mam....I

should have paid more attention to

your question. (Nivedita gets anxious,

looks at the glass of water kept on the table)M2: That’s ne. Have some water

and then answer my question.

Nivedita:  (after thinking for few

seconds)  Thank you Mam! Mam,

Ramanuja propounded Vishisht advait

while Madhava talks about Dvait vada.

According to Ramanuja, Brahma and

God are identical, God is accepted as

the highest reality. Madhava ‘s theory

believed that God or Brahma is an

independent reality while achitta ormatter is dependent reality, though

different from Brahma, is dependent

on Brahma. But for Ramanuja, achitta

is that part of Brahma through which

God has created this world.

(M2 looks satised with the answer)

M2: What is Hume’s theory of

ideas and impression?

Nivedita: Mam may I use this pen

and paper.... (pointing at the table)

M2: Yes, you may.

Nivedita:  Mam, according to

Hume the source of valid knowledge

is experience. Through experience we

gain perception. According to Hume

perception are of two types –

Impressions and Ideas. Impressions

are the direct object of our knowledge

so they are lively...... separate and

independent of each other. They cannot

Page 61: CA June 2016 Eng Xaam.in

8/16/2019 CA June 2016 Eng Xaam.in

http://slidepdf.com/reader/full/ca-june-2016-eng-xaamin 61/68

Debate

No problem can withstand the assault of sustained thinking. ~ Voltaire

 Just as there are two sides to a coin, there are two sides to every story (and even more at times). Sometimes,

there is nothing called the truth; there are only viewpoints. Depending on which side of the fence you

stand, you could be a proponent or an opponent, you could be a supporter or you could be a critic,

you could be in favour of the motion of debate or you could be against it.

Life is full of debates because you want to question and, in turn, have to be receptive and open to

questions and probes. You are in constant debate with yourself and with others. You are in search oftruth and truth is never nal; the search of truth is a continuous exercise.

An 'answer' you write in the UPSC CSE Examination or give as a reply in the concluding Personality

test should reect that truth. But truth, as we surely know, is not absolute. There are views, perspectives,

sides, dimensions, frames of reference, vantage points to every answer. A rational mind should

acknowledge this and try to nd and grasp the 'whole' truth.

When UPSC says in the CS examination notication that it seeks candidates who possess intellectual

qualities, social traits, balance of judgement, clear and logical exposition, variety and depth of interest and so

on, it clearly underscores that it needs a balanced personality who would be a t candidate to be acivil servant capable of considering competing viewpoints in any situation in the district or in the

secretariat. Debate brings out such qualities in a human being. It reveals what you have been ignoring.

It tells you things you ordinarily miss, it shows you your narrowness and one-sidedness; it makes

you more considerate, reasonable and complete. That is why it is said that you learn more from those who

disagree with you than from those who agree with you and that a wise enemy is better than a foolish friend

because (s)he teaches you more.

"Begin challenging your own assumptions. Your assumptions are your windows on the world.

Scrub them off every once in a while, or the light won't come in." With this in mind, we bring to youthis Debate section of the magazine where you can learn more about a topic and your own ignorance.

Knowledge and information is never complete; you need a debate to nd out more about things and

UPSC will try to nd out if you are balanced enough and know all sides of story (through queries lie

'analyze critically' or 'examine'  in Mains paper or through direct questions in the Personality Test).

Read this section with an open mind. Because what you know is not necessarily what there is all that

could be known; there is always more. And make it a habit to seek more knowledge, to talk to people

who hold different views and once in a while shake hands with your enemies. The world needs debates,

not wars. Start a debate.

Page 62: CA June 2016 Eng Xaam.in

8/16/2019 CA June 2016 Eng Xaam.in

http://slidepdf.com/reader/full/ca-june-2016-eng-xaamin 62/68

Drishti Current Afffairs Today || June 2016 || 223

FOR

–Amit Singh

 What is AADHAR?

Aadhar is a 12 digit uniqueindividual identication numberwhich can be used as proof of identityand address, anywhere in India. Anyperson who has resided in India for182 days or more, in one year beforethe date of application, is eligible forenrolment for Aadhar.

Despite being a potential tool tocurb leakages in subsidies and services,there have been a lot of apprehensionsabout the Aadhar on certain grounds.Some of them are discussed below.

Is Aadhar mandatory foravailing services provided bythe government?

Supreme Court in March 2014ruled that “no person shall be deprivedof any service for want of Aadhar numberin case he/she is otherwise entitled/ eligible”. Contrary to this, the bill textallowed government to make Aadharauthentication compulsory for old agepensions, train bookings, marriagecerticates, for getting driving licence,buying a SIM card etc.

Is Aadhar the onlyway to curb leakages?

There are different types of

leakages in the delivery of subsidies

and services. Aadhar may be able tocurb only specic types of leakages,mainly those related to ghostbeneciaries. But for leakages such asthose in Public Distribution System(PDS) where middle men and rationshop owners collude to divert theentitlement into the market illegally,theidea of ‘identity fraud’ gains littleprominence.

The claim that Aadhar will,

by itself curb the leakageneeds chosen attention.

Poor internet connectivity in ruralareas, lack of electricity, technicalmalfunctioning of systems can lead toauthentication failures. It is to be notedthat poorer states where PDS is mostneeded are least prepared forinfrastructure required for BiometricsAuthentication Linking. Aadhar basedauthentication to PDS entitlement in

such states will do more harm thangood.

Convergence of theresident’s information.

As of now, information on eachresident is available only in silos thatare structurally separated. And itrequires huge effort to link informationin one silo to another. For example:By accessing someone‘s driving licensenumber one may be able to get details

about his permanent address, but

fetching his banking details or say hiscall details will be extremely difcult.

However, in case of Aadhar aunique number links information ineach silo. So once you get hold ofAadhar number of somebody, theinformation related to banks account,phone calls, permanent address, canall be fetched either malade or as asecurity breach.

Is ‘Right to Privacy’ afundamental right?

In 2015, Government hadcommunicated its stand to the SupremeCourt and claried that privacy wasnot a fundamental right andconsequently no fundamental right isbreached by Aadhar. Since many ofthe earlier judgments afrmingprivacy as a fundamental right, weredelivered by smaller benches, the courtagreed to refer to this question by a

larger constitutional bench.Proponents of Aadhar fail to

answer why the Aadhar bill could notwait for the judgment by the largerconstitutional bench.

Moreover dissent is the bedrockof democracy. And right to privacy isthe foundation of the freedom todissent. How many of us would liketo participate in an open protest, inwhich furnishing all our details (phone

DOES AADHAR INVADE PRIVACY TO

 ACHIEVE FISCAL CONSOLIDATION?The introduction of the Aadhar bill and consequently its passage has evoked mixed reactions from different stakeholders.

The proponents of this bill cite its multidimensional benets and consider it a tool for attaining nancial consolidation. On theother hand, the critics doubt its motives and argue over the manner of its passage, privacy concerns and its potential for use asa tool for mass surveillance in future. The following debate tries to unravel these issues and much more.

The Aadhar (Targeted Delivery of Financial and Other Subsidies, Benets and Services) Act, 2016 intends to provide fortargeted delivery of subsidies and services to individuals residing in India.

Page 63: CA June 2016 Eng Xaam.in

8/16/2019 CA June 2016 Eng Xaam.in

http://slidepdf.com/reader/full/ca-june-2016-eng-xaamin 63/68

Drishti Current Afffairs Today || June 2016 || 229

SUPPLEMENT

ART & CULTURE

OF

INDIA 

More than 60 Pages...

RESEARCHED,

Easy to Understand...

Page 64: CA June 2016 Eng Xaam.in

8/16/2019 CA June 2016 Eng Xaam.in

http://slidepdf.com/reader/full/ca-june-2016-eng-xaamin 64/68

Supplement : Ar t and Culture

Drishti Current Afffairs Today || June 2016 || 235

The Mathura School of Art 

  This school of art ourished in theareas around the banks of the riverYamuna in the period roughly fromthe 1st century AD to the 3rd centuryAD.

  The Mathura sculptors primarilyused red sandstone quarried in thenearby mines of Sikri.

  The art style closely followed theYaksha images of the earlierMauryan times and inuence ofBuddhism, Hinduism and Jainismcould be seen.

This school of art is known foriconographic symbolism, forexample the Hindu deities wereimaged according to their avayudhas

(for example association of Shivawith the linga).

In Buddhist art, the Buddha isdepicted with a larger halo than inthe Gandhara school and twoprominent Bodhisattvas were builtknown as Padmapani (holding thelotus) and Vajrapani (holding thethunderbolt).

 Yaksha

  Buddha is depicted wearing atransparent dhoti which is pulledabove His shoulders. He is eitherbald or curly haired and is mostlyseated under a pipal tree.

  Buddha is often anked by twobodhisattvas one on either side, or

by two smaller gures of Indra andBrahma.

  Both Buddha and Jain Tirthankarimages carry the auspicious markurna on their foreheads.

  Shiva is sometimes depicted as the

Ardhanarishvara (half man halfwoman), the Harihara (Vishnu andS hi v a c o m b i ned ) a nd theChaturvyuha Shiva (Shiva’s threeemanations).

  Mathura art was very popular inits time and was exported to placessuch as Kaushambi, Sarnath andAhichchhatra (near modern dayRamnagar, Uttarakhand).

The Amaravati School of Art

This school of art ourished in SouthIndia (Andhra region) in the areassurrounding the banks of the riverKrishna.

The patron kings of this school werethe Satvahana kings.

This school of art is famous for thereliefs (sculpture carved on a surfacee.g. wall) and these reliefs areconsidered one of the world’s bestnarrations on sculpture.

  These reliefs were carved upon thecharacteristic green-white limestonefound in the Andhra area.

  Also, the use of white marble ischaracteristic of this school of art.

The ‘tribhanga’ posture is a recurringtheme while major emphasis wason narrative and dynamic images.

  The Buddha is sometimes depictedin both iconic and aniconic (non-human, non-animal image) formswhere image of the Buddha is iconic

and the accompanying ‘emptythrone’ is aniconic.

Architecture of the Gupta Age

From the 4th century onwards,the Gupta Empire came into being.This period is called the ‘Golden Ageof Indian Art & Architecture’.

Under the patronage of the earlierGupta rulers, who were Buddhists,cave architecture continued to develop.

Ajanta caves, in Maharashtra weremade as Viharas and Chaityasduring the period 200 BC to 650AD. It was built by cutting the rocksof the Inhyadri Hills of the Sahyadriranges. The inscriptions in these

caves are Buddhist inscriptionsmade under the patronage ofHarishena, a Vakataka ruler. Thegures on the walls were done usingthe fresco style of painting whichdepicted the Jataka Tales along withimages of animals and plants, in anaturalistic way. Colors used tomake these paintings were createdusing vegetables and minerals. Thecolor blue was not used anywherein these paintings while the outline

of the paintings were done in thecolor red. Both the Chinese traveler,Fa Hien and Hieun Tsang mentionsthe Ajanta caves in their writings.

  Ellora caves, also in Maharashtrawere made for Buddhist, Jain andBrahmanical religious work. Thesewere made between 5th and 11thcentury AD. The caves are of diversestyle in its theme and dedication.For example there is a Chaitya cavededicated to Vishwakarma (caveno. 10), a cave dedicated as Kailashtemple (cave no. 16; made by aRashtrakuta king) and two Jaincaves known as Indra Sabha and

 Jagannath Sabha. Cave number 14and 15 are called ‘ravan ki khai’and Dashavatar caves respectively.

  Bagh caves, in Madhya Pradesh,built around 6th century AD are agroup of 9 Buddhist caves.

   Junagadh caves, in Gujarat, have a

citadel called Upar Kot in the prayerhall.

  Nasik caves, in Maharashtra are agroup of Buddhist caves known asPandav Leni. There are no imagesof Buddha here because the patronsof the cave belonged to the Hinayana

school of Buddhism. Instead ofimages there are symbols of theBuddha like his throne andfootprints.

Page 65: CA June 2016 Eng Xaam.in

8/16/2019 CA June 2016 Eng Xaam.in

http://slidepdf.com/reader/full/ca-june-2016-eng-xaamin 65/68

Supplement : Ar t and Culture

236 || Drishti Current Afffairs Today || June 2016

Under the patronage of the laterGupta rulers who were Hindu, templearchitecture ourished.

  Temple architecture began as simple

temples with at roofs, shallow

pillars, and low platforms and was

generally square shaped. Later on

this evolved into more sophisticated

styles with the temples being placed

on higher platforms having variousshapes and with shikharas as roofs.

Nagara style of temples began to

be built from 5th century onwards.

This style is called the Panchayatan

s tyle . The most important

characteristics of this style are the

use of multiple shrines to surround

the main shrine. There are also

mandaps (hall), pillars and shikharas

in the design of such temples. But

no water tanks were built in the

temple premises.

The Nagara School is FurtherSub-Divided into–OdishaSchool, Khajuraho School andSolanki School.

The Odisha School

Lavishly decorated exterior walls,interior walls are simple and plain,pillars absent from the verandah, useof iron girders to make the roof, templebuilt on square platform, templesprotected by boundary walls,shikharas are called ‘rekhadeuls’,mandaps are called ‘jagmohanas’.Examples of this school – Sun Templeat Konark (also called Black Pagoda), Jagannath Temple at Puri and LingarajTemple at Bhubaneswar.

Types of Shikharas

  Rekha-Prasad (Latina) shikhara– these have a square base uponwhich the walls curve inside andmeet at a point on top.

  Phamsana – these were built witha bigger base but were shorter inheight than the Rekha-Prasad

shikhara. These generally moveupwards in a slope owing in astraight line.

  Valabhi – these come withrectangular bases with roofshaving vaulted chambers.

The Khajuraho School

Built by Chandel kings, also called

Chandel School. Both interior andexterior walls were lavishly decorated,

Name andLocation of Caves

Period Number andType of Caves

Features and aspects

Ajanta Caves

Near Aurangabad,Maharashtra

Between 200 BCto 650 AD

Out of 29 caves, 25 wereViharas and 4 wereChaityas.

  Painted and inscribed by Buddhist monksunder the patronage of Harishena.

  Fresco paintings done with vegetable colours.

Blue colour was not used. Themes were based on the life of Buddha and

the stories from the Jataka tales.

Ellora Caves

Near Aurangabad,Maharashtra

Between 5thand 11thcentury AD

Out of 32 caves, 16 wereBrahmanical, 12 wereBuddhist and 4 were Jain.

  Painted by artists of various guilds fromKarnataka, Tamil Nadu and Vidarbha.

  The themes are diverse.

  The architectural style is different from AjantaCaves and only Ellora Caves had courtyards.

Bagh Caves

Near the bank of Baghriver, Madhya Pradesh

Around 6thcentury AD

All 9 caves wereBuddhist.

  Built in the architectural likeness of the AjantaCaves.

  Some of the cave paintings made here belong

to the Gupta age. Junagadh Caves,

Gujarat

Between 1st to4th century AD

3 groups of numerousBuddhist caves.

  Baba Pyare Caves, Khapra Kodia Caves andUpar Kot Caves.

  A 30-50 ft high citadel in Upar Kot to markthe prayer hall.

Nasik Caves (PanduLeni), Maharashtra

Between 3rdcentury BC to2nd century AD

23 Buddhist Caves.   Built under Hinayana sect there are no imagesof the Buddha.

  Buddha is represented by motifs and symbols,for example – footprints and throne.

Mandapeshwar Caves(Montperir Caves),

Maharashtra

8th century AD Originally a Brahmanicalcave it was later changed

into a Christian cave.

  It has sculptures of Nataraja, Brahma and Vishnu.

Page 66: CA June 2016 Eng Xaam.in

8/16/2019 CA June 2016 Eng Xaam.in

http://slidepdf.com/reader/full/ca-june-2016-eng-xaamin 66/68

Supplement : Ar t and Culture

284 || Drishti Current Afffairs Today || June 2016

  1. Which of the following showed the greatest uniformityin Harappan settlements?

  (a) Town planning (b) Building

  (c) Bricks (d) Religious practices

  2. Which metal has so far not been discovered in Harappansites?

  (a) Copper (b) Gold

  (c) Silver (d) Iron

  3. Consider the following statements regarding the IndusValley Civilisation

1. It was predominantly a secular civilisation andthe religious element, though present, did notdominate the scene.

  2. During this period cotton was used formanufacturing textiles in India.

Which of the statement(s) given above is/are correct?  (a) Only 1 (b) Only 2

  (c) Both 1 and 2 (d) Neither 1 nor 2

  4. Which one of the following is not the characteristicsfeature of the Harappan settlements?

(a) Doorways and windows generally faced the sidelanes and rarely opened into the main streeets

  (b) Houses generally had separate bathing areas andtoilets

  (c) The citadel was walled but the lower towns wasnot walled

  (d) Drains and water chutes from the second storey

were often built inside the wall

  5. In the Harappan period, seals were normally made of

  (a) lead (b) gold

  (c) silver (d) steatite

  6. The scripts of the Indus Valley Civilisation was

  (a) Boustrophedon, still not satisfactorily deciphered

  (b) Brahmi

  (c) Nagri

  (d) None of these

  7. Match the following

  List-I List-II

  A. Ghaggar 1. Mohenjodaro

  B. Ravi 2. Kalibangan

  C. Indus 3. Lothal

  D. Bhogava 4. Harappa

  Codes:

  A B C D

  (a) 1 2 3 4

  (b) 3 4 1 2

  (c) 3 1 4 2

  (c) 2 4 1 3

  8. Which one of the following statements regardingHarappan Civilisation is correct?

(a) The standard Harappan seals were made of clay

  (b) The inhabitan ts of Harappa had neither

knowledge of copper nor bronze  (c) The Harappan Civilisation was rural based

  (d) The inhabitants of Harappa grew and used cotton

  9. Which one of the following sites of the Indus ValleyCivilisation had an ancient dockyard?

  (a) Kalibangan (b) Lothal

  (c) Rangpur (d) Harappa

  10. The site of Harappa is located on the bank of river

  (a) Saraswati (b) Indus

  (c) Beas (d) Ravi

  11. Which of the following was conspicuous by its absence

in the terracottas of Indus Civilisation?  (a) Sheep (b) Buffalo

  (c) Cow (d) Pig

  12. Which one of the following was not known to theHarappans?

  (a) Construction of wells

  (b) Construction of pillars

  (c) Construction of drains

  (d) Construction of arches

  13. Match List I with List II and select the correct answerusing the codes given below the lists.

  List I List II  A. Lothal 1. Ploughed eld

  B. Kalibangan 2. Dockyard

  C. Dholavira 3. Terracotta replica of a plough

  D. Banawali 4. An inscription comprising ten

  large sized sign of the Harappan

script

  Codes:

  A B C D

  (a) 1 2 3 4

  (b) 2 1 4 3

  (c) 1 2 4 3  (d) 2 1 3 4

  14. A copper chariot of Harappan times was discovered at

  (a) Kuntal (b) Rakhigarhi

  (c) Daimabad (d) Banawali

15. Which one among the following is not true with regardto Rigveda Samhita?

  ( a) There are about 300 non-Indo-European wordsin Rigveda

  (b) There is a reference to Dasarajna (battle of tenkings) in the Rigveda

100 ART AND CULTURE MCQs

Page 67: CA June 2016 Eng Xaam.in

8/16/2019 CA June 2016 Eng Xaam.in

http://slidepdf.com/reader/full/ca-june-2016-eng-xaamin 67/68

Page 68: CA June 2016 Eng Xaam.in

8/16/2019 CA June 2016 Eng Xaam.in

http://slidepdf.com/reader/full/ca-june-2016-eng-xaamin 68/68